Vous êtes sur la page 1sur 37

AAA

BRIGHT ACADEMYTM

CENTRE FOR EXCELLENCE IN EDUCATION

SBI MOCK TEST-1


7.

REASONING

Who is the immediate left of M:


a. K
b. W

c. D
d. T
e. None of these
In a certain code GEAR is written as 5934 and
RIPE is written as 4869. How is PAGE written in Directions (8 12): In each of the questions below are
that code:
given three statements followed by three conclusions
a. 6359
b. 6539
numbered I,II, III . You have to take the given statements to
be true even if they seem to be at variance from commonly
c. 4359
d. 6459
e. none of these
known facts. Read all the conclusions and then decide
Directions (2 3) : Read the following information carefully which of them given conclusions logically follows from the
given statements disregarding commonly known facts:
and answer the questions which follow:
1.

2.

8.
(i)
P x Q means P is brother of Q
(ii)
P Q means P is sister of Q
(iii)
P Q means P is mother of Q
(iv)
P + Q means P is father of Q
Which of the following means D is maternal uncle
of K:
a. D N K
b. D N + K
c.

3.

DxNK

d. D x N + K

e. none of these

Which of the following means M is nephew of R:


a. M x T + R
b. R J + M x T
c. R J + M

d. R x J + M

b. only II follows

e. none of these

Directions (4 7) : Study the following information


carefully and answer the questions given below:
M, D, P, K, R, T and W are around a circle facing at the 9.
centre. D is second to the right of P who is third to the right
of K. T is third to the right of W who is not an immediate
neighbour of D. M is third to the left of R.
4.

Who is second to the right of T:


a. D
b. K
c. M

5.

e. none

In which of the following pairs is second person


sitting to the immediate right of the first person:
a. DT
b. TP
c.

6.

d. Data inadequate

PR

d. KW

e. None of these

Who is to the immediate left of R:


a. W
b. P
c.

d. T

e. none of these

Statements:
All pins are rods
Some rods are chains
All chains are hammers
Conclusions:
I.
Some pins are hammers
II.
Some hammers are rods
III.
No pin is hammer
a. Only I follows

c.

only III follows

d.

only either I or III and II follow

e.

none of these

Statements:
Some books are papers
Some papers are desks
Some desks are chairs
Conclusions:
I.
Some books are desks
II.
Some papers are chairs
III.
Some books are chairs
a. None follows
b. only I follows
c. only II follows

d. only III follows

e. only I and II follow


10. Statements:
Some pots are buckets
All buckets are tubs
All tubs are drums
Conclusions:
I.
Some drums are pots
II.
All tubs are buckets

Page | 1

III.
Some drums are buckets
a. Only I and II follow
b. only I and III follow

Give answer (d) if neither conclusion I or conclusion II


follows

c.

only II and III follow

Give answer (e) if both conclusion I and conclusion II


follow

e.

none of these

d. all follow

13. Statements:
H T , T % M, M F
Conclusions: I. F $ T
II. H M
14. Statements:
B @ N, N T, T $ K
Conclusions: I. T @ B
II. T $ B
15. Statements:
R$J,JF, F%H
Conclusions: I. H J
II. R $ F
16. Statements:
JD ,D@N,N%F
b. only III follows
Conclusions: I. J F
II. F D
17. Statements:
c. only either I or III follow
BT, T$H,H@M
Conclusions: I. M T
II. B H
d. only either I or II and III follow
18. Consider the following sentences:
Nowadays the FMCG companies woo their customers
e. none of these
or buyers by selling their goods or products in small
sachets. This has increased the sale of the product .
12. Statements:
Which of the following may be the appropriate
Some buses are trucks
reason for the given facts:
Some trucks are boats
a. People generally want to buy any products in
No boat is jeep
small quantity
Conclusions:
b. FMCG companies save cost of packaging and so
I.
Some jeeps are buses
their profit is maximized
II.
Some boats are buses
c. It is easy to store any product in sachets so buyer
III.
Some jeeps are trucks
prefer them.
a. None follows
b. only I follows
d. It is possible to assess the quality of products
c. only II follows
d. Only III follows
without wasting much money when a sachets is
bought.
e. only II and III follow
e. Generally people consume the whole quality of
product contained in the sachets at once and so
Directions (13 17): In the following questions, the
people buy a large number of sachets at one time.
symbols @, , $ , % and are used with the following
meaning as illustrated below:
Directions (19 - 23): Study the following information
carefully and answer the given questions:
P Q means P is either greater than or equal to Q
In a certain code language:
P % Q means P is either smaller than or equal than Q
demand and supply market is written as pa ni le re
P @ Q means P is neither smaller than nor greater than
Q
market needs more demand is written as de ja ni le
11. Statements:
All pins are bags
All chalks are bags
All needles are bags
Conclusions:
I.
Some needles are pins
II.
Some chalks are needles
III.
No needle is pin
a. Only I follows

P $ Q means P is smaller than Q

supply demand is related is written as le fe re ab

P Q means P is greater than Q

more related to economics is written as ka ha ab ja

Now in each of the questions assuming the given statement 19. What is the code for economics:
to be true, find which of the two conclusions I and II given
a. ab
b. ka
below them is/are definitely true:
c. ha
d. ja
e. either ka or ha
Give answer (a) if only conclusion I follows
20. Which of the following represents supply related
Give answer (b) if only conclusion II follows
market:
a. ab ni de
b. ni re ab
Give answer (c) if either conclusion I or Conclusion II
follows
c. pa ni re
d. de ab ni
e. none of these

Page | 2

21. What is the code for more:


a. fe
b. ni
c. de

d. ja

e. cant determine

26. Which of the following represents the positions of


pays in the step II:
a. Seventh from right
b. sixth from left

22. Which of the following may represent market


needs more customers:
a. fe ja ni sa
b. de ja ni pa
c. ni ja ka pa

c.

eight from right

e.

Fifth from left

d. fourth from left

27. Which word/number would be at the seventh


position from the right in the step IV:
a. Work
b. run

d. pa ni fe re

e. le re ni ja
c.

23. What is the code for needs:


a. ni
b. fe
c. pa

d. le

e. none of these

Input :
plan more vacation 35 56 92 nice holiday tours 84 61 12
Step I:
92 plan more vacation 35 56 nice tours 84 6112 holiday
Step II:
92 84 plan vacation 35 56 nice tour 61 12 more holiday
Step III:
92 84 61 plan vacation 35 56 tour 12 nice more holiday
Step IV:
92 84 61 56 vacation 35 tours 12 plan nice more holiday
Step V:
92 84 61 56 35 vacation 12 tours plan nice more holiday
Step VI:
92 84 61 56 35 12 vacation tours plan nice more holiday
And step VI is the last step of the rearrangement as the
desired arrangement is obtained.
As per rules followed in the above steps, find out in each of
the questions the appropriate step for the given input.
Input: hard work pays 96 42 in 79 long run 18 25 57
24. Which step number is the following output:
96 79 57 42 work run 18 25 pays long in hard
a. Step II
b. Step III
c.

Step IV

d. Step V

d. 18

e. 25

28. Which word/number would be at the ninth position


from the left in the Step V:
a. Long
b. 18

Directions (24 28): Study the following information


carefully and answer the questions given below:
A word and number arrangement machine when given an
input line of words and numbers rearranges them following
a particular rule in each step .The following is an illustration
of input and various steps of rearrangement .(all the
numbers are two digit numbers).

long

c.

pays

d. 25

e. in

Directions (29 35): Study the following information


carefully and answer the given questions:
Eight persons J,K,L,M,N,O,P and Q are sitting around a
circular table facing the centre, but not necessarily in the
same order.

P is third to the left of M.


O is second to the left of P.
Only one person sits between M and L.
28 year old is neighbour of both O and P.
K is third to the left of 28 years old.
Q is second to the right of J.
39 year old second to the left of N.
13 year old is second to the right of K.
K is elder to N but younger than L. L is not the eldest.
M is 16 year old.
16 year old and 9 year old are not immediate
neighbours.

Ages are as 9, 13, 16, 18, 21, 28, 33, and 39


29. Who amongst the following is sitting exactly
between M and L:
a. Q
b. nine-year old
c. 18 year old

d. P

e. 13 year old

30. Which of the following pairs represents the


immediate neighbours of K:
a. 16 year old and J
b. 9 year old and Q
c.

M and 18 year old

e.

L and 9 year old

d. M and 28 year old

31. Which of the position of J with respect to the


position of P:
a. Fourth to the left
b. fourth to the right
25. How many elements (words/numbers) are there
between work and in as they appear in step III:
c. Fifth to the left
d. Third to the left
a. Six
b. five
e.

there is no such step

c.

seven

d. four

e. three

e. Third to the right

Page | 3

32. Starting form J, if all the persons are made to sit in


the alphabetical order in clockwise direction, the
positions of how many (excluding J) will remain
unchanged:
a. None
b. one
c.

two

d. three

e. more than three

33. Who among the following is sitting second to the


right of P:
a. O
b. 9 yr old
c.

33 yr old

d. M

e. N

34. Who among the following is sitting just opposite to


O:
a. L
b. N
c.

d. J

e. P

35. Who among the following is third to the left of 21


yr old:
a. N
b. Q
c.

d. L

e. M

ENGLISH LANGUAGE

has limitations relating to infringing the jurisdiction of


states. It is could have aggregated and disseminated
experiences and knowledge, stocked many of the essential
items required in an emergency or worked with agencies to
ensure sufficient stocks, but hasnt.
While the reaction to major disasters is dismal, the
response to emergencies like accident is equally sad.
Victims lie unattended since passersby are wary of getting
caught in a labyrinthine of police and legal systems. The
resulting delay in treatment converts injuries into deaths. Of
late, unique had free service to provide assistance in
emergency cases is operational. Emergency Management
and Research institute (EMRI) is a professionally managed
operation initiated by the vision and grant from Ramalinga
Raju. The service, which is a successful example of public
private partnership likely to become operational in a few
states in the near future. Given the sad failure of
conventional government organizations in handling
disasters, it is time we looked at the PPP model as an
alternative without the government seeking in any way to
abdicate its responsibility. While the state provides the
funding, private organizations will provide the drive,
professionalism, competent management and output linked
efficiency of a good corporate organization. Combining the
sensitivity and purpose of an NGO with private
entrepreneurial drive to handle disasters together is thus a
worthwhile challenge for both corporate and the
government.

Directions (36 44): Read the following passage carefully


and answer the questions given below it. Certain words are
printed in bold to help you locate them while answering 36. Why do bystanders not help accident victims:
a. They lack the necessary medical knowledge.
some of the questions.
b. It is the responsibility of the driver of the vehicle.
c. They are wary of cumbersome police formalities
We have witnessed several disasters in recent times, some
and legal systems.
natural others man made. The frequency of such
d. Medical attention via satellite phones can be
calamities has injured us and deadened our collective
provided promptly.
sensitivity, but that does our collective sensitivity, but that
e. None of these
personal tragedy of each victims family and community.
The economic loss is only secondary to the human 37. Which of the following is not true in the context of
the passage:
suffering, but is also substantial. The government whether
a. Manmade disasters occur more frequently than
State or central has standardized its response. This
natural disasters.
consists of reacting late, blaming others, visits by VIPs
b. The public private participation model has been
announcing a relief package including compensation for
successful in handling emergencies
those affected and then forgetting all about it. There seems
c. Floods occur every year in some Indian states.
to be little attempt at drawing lessons from each disaster,
d. Analysis of previous disasters will help us cope
storing the knowledge for future use, long term planning for
with future ones.
possible pre-emptive action. Preparedness for disasters
e. People have become indifferent to disasters.
thus falls short of what is possible using todays
38. What is the authors view on the governments
technologies.
current reaction to natural disasters:
Floods in many parts of India like the states of Bihar and
a. The government has not been able to handle
Assam are a yearly phenomenon. Yet the government
disasters and should seek foreign aid.
seems to be caught by surprise year after year. It is obvious
b. A central government agency should be set to
that tarpaulins, vaccines, medicines, clothes, satellite
speedup coordination in rescue efforts.
phones, large numbers of doctors and paramedical staff
c. It has failed to utilize donations effectively to
etc. will be needed as will boats and buses for evacuation.
provide relief.
This is known to all those who have combated emergencies
d. The government is apathetic and has not
yet the non availability of these essential services and
managed to handle disasters effectively
commodities occurs. Worse the organizational structure
e. None of these
and mechanisms for dealing with disasters are lethargic 39. Why is there a lack of medical care at disaster
and ill defines. The national disaster management Agency,
sites:
set up a short time ago being a central government agency
a. Inadequate transportation facilities

Page | 4

b. Lack of disaster management training for medical


c. leading into
d. have lead to
staff.
e. no correction required
c. Loss of medical supplies due to dangerous
conditions.
d. Safety of medical staff due to dangerous 47. Public health facilities are more scarce since patients
have to wait for hours to see a doctor.
conditions.
a. Too scarce for
b. very scarce because
e. None of these
40. What does the author consider a worthwhile
c. so scarce that
d. quite scarce that
challenge for both corporate and the government:
a. Government should gradually transfer disaster
e. no correction required
management to corporate.
b. Their working together to manage disasters 48. If he is fortunately to be successful , you should give
completely keeping public interests in mind.
something back to society.
c. Reducing the incidence of manmade disasters.
a. Unless you are fortunate
d. Mitigating the financial losses sustained during
b. If you are fortunate
natural disasters
c. By being fortunate
e. None of these
d. If you were fortunately
e. no correction required
Directions (57-58): Chose the word which is most similar
49. Last week the committee summarily rejected their
in meaning to the word printed in bold as used in the
proposal for acquisition of the investment bank.
passage.
a. Have rejected in summary
b. Summarily reject
41. Infringing
c. Is rejected summarily
a. Breaking
b. violating
d. Were summarily rejected
e. no correction required
c. provoking
d. hampering e. disobeying
42. Frequency
a. Sound
c.

recurrence

b. habit
d. average

e. occasion

Directions: (59 60): choose the word which is most


opposite in meaning to the word printed in bold as used in
the passage:
43. Lethargic
a. Healthy
c.

favorable

44. Dismal
a. Depressing
c.

competent

b. active
d. awake

e. intense

b. upset
d. animated

e. smiling

Directions (50-52): Rearrange the following sentences (A),


(B),(C),(D) ,(E) and (F) in the proper sequence to form a
meaningful paragraph, then answer the questions given
below them.
A. As a result the non-stop tensions and anxieties at
work often result in health related problems.
B. The truth is we cannot change the world of work
C. We spend at least half our waking hours at work
D. We have therefore to take charge and transform
the way in which we respond to our work
environment
E. So how can we control these problems and perform
at work?
F. However we can change the way we feel and deal
with various situations.
50. Which of the following should be the last(sixth)
sentence after rearrangement:
a. B
b. C

Directions (45-49): Which of the phrases (a),(b),(c),(d)


given below each sentence should replace the word/phrase
c. D
d. E
e. F
printed in bold in the sentence to make it grammatically
correct? If the sentence is correct as it is given and no 51. Which of the following should be the Fifth
correction is required, mark (e) as the answer
sentence after rearrangement:
45. Which impressed me most about him was that his
commitment to his work as well as his family.
a. He was committed
b. that he is committed
c. he has committed for

d. his commitment to

e. no correction required
46. It is the crippling losses suffered by US banks have
led to the current economic slump.
a. Which have led to
b. that have led

a. B

b. C

c.

d. E

e. F

52. Which of the following should be the Second


sentence after rearrangement:
a. A
b. B
c. C

d. D

e. E

Page | 5

Directions (53 - 55): Read each sentence to find out


whether there is any grammatical error or idiomatic error in
it. The error, if any, will be in one part of the sentence. The
number of that part is the answer. If there is no error, the Directions (66 70): Study the following pie-chart carefully
to answer the questions that follow:
answer is (e). (Ignore errors of punctuation, if any)

QUANTITATIVE APTITUDE

53. These companies have been asked (a)/ to furnish their Degree wise breakup of expenditure of a family in a
financial details (b)/ and information about (c)/ its month total amount spent in a month is Rs. 45,800.
board members. (d)/no error (e)
54. The scheme which will be launched (a)/ during the
next two years (b)/ require an additional investment
(c)/of one hundred crores. (d)/ no error (e).
55. Road developers unable (a)/to complete their projects
(b)/ on time will not be (c)/awarded new ones.(d)/ no
error (e).
Directions (56 90): In the following passage there are
blanks, each of which has been numbered. These numbers
are printed below the passage and against each. Five
words are suggested, one of which fits the blank
appropriately. Find out the appropriate word in each case.
Recently the World Bank and the Asian Development Bank 66. What is the amount spent by the family on
commuting:
(ADB) 56 separate reports on poverty. The World Bank
a. Rs. 10,076
b. Rs. 10,534
Report 57 its benchmark of extreme poverty by 25 cents
from $ 1 per person per day to $ 1.25 per person a day.
c. Rs. 6,870
d. Rs.8,702
The ADB announced an ever 58 benchmark of $ 1.35 per
person a day. These new benchmarks are 59 on surveys in
e. none of these
the worlds poorest countries.
67. What is the respective ratio of amount spent by
Experts often like to 60 that poverty has declined because
family on Medicine to the amount spent of
of economic growth in India and China. This is wrong and
Groceries:
61 .In the past twenty five years the poverty rate in India
a. 1:2
b. 12:21
has 62by less than one percentage point a year. 63 we use
a poverty line of $1 per person per day or $ 1.25 per person
c. 3:5
d. 11:23
e. none of these
per day makes little 64. The number of poor in India is
large. The purpose of thee statistics is not to dispute them 68. What is the total amount spent by the family on
but to 65 whether the benefits for economic growth are
entertainment and shopping together:
being shared with the poor.
a. Rs. 9,618
b. Rs. 13,282
56. a. declared
c. print
57. a. heightened
c. raised
58. a. better
c. plausible
59. a. based
c. inferred
60. a. realize
c. discover
61. a. adverse
c. corrupt
62. a. deplete
c. declined
63. a. unless
c. instead
64. a. difference
c. contract
65. a. acknowledge
c. care

b. released
d. issue
b. announced
d. maintained
b. significant
d. higher
b. collected
d. derived
b. claim
d. recommend
b. opposing
d. rejected
b. plunge
d. weaken
b. despite
d. whether
b. effect
d. question
b. suggest
d. inspire

e. publish

c.

Rs. 13,740

d. Rs. 11,908

e. none of these
e. notified
e. lower
e. gathered

69. Total amount spent by the family on Groceries,


Entertainment and investments together forms
approximately what per cent of amount spent of
commuting:
a. 209
b. 76

e. criticize
c.
e. misleading
e. fell
e. regardless
e. option
e. study

154

d. 42

e. 218

70. Amount spent by the family on Medicine forms


what
percent
of
amount
spent
on
Shopping:(rounded off to two digits after decimal):
a. 43.67
b. 49.52
c.

57.89

d. 61.89

e. none of these

Directions (71 75): study the following graph carefully to


answer the questions that follow:

Page | 6

Total number of boys and Girls in five different 76. 63215 x 82 = ? x 42105
departments
a. 101
b. 123
Boys

c.

Girls
77.

300
280
260
240
220
200
180
160
140
120
100
80
60
40
20
0

147

d. 165

=?
a. 240

b. 270

c.

d. 290

330

78. (. ) =?
a. 3000
c.

3500

e. 189

e. 310

b. 3300
d. 3700

e. 3900

Directions (79-82): What should come in place of the


question mark (?) in the following number series:
79. 20 24 33 49 74 110 ?
a. 133
b. 147
c.

159

d. 163

e. 171

80. 16 24 48 120 360 1260 ?


a. 3780
b. 4725
71. The number of girls from Biology department is
c. 5355
d. 5040
approximately what per cent of the total number of
girls from all the departments together:
81. 8 31 122 485 1936 7739 ?
a. 32
b. 21
a. 30950
b. 46430
c.

37

d. 43

e. 27

c.

34650

d. 42850

e. 4410

e. 38540

72. What is the difference between the total number of 82. 499 622 868 1237 1729 2344 ?
boys and the total number of girls from all the
a. 3205
b. 3082
departments together:
a. 440
b. 520
c. 2959
d. 3462
e. 2876
c.

580

d. 460

e. none of these

Directions (83-85): What should come in place of the


question mark (?) in the following questions?

73. What is the average number of boys from all the


departments together:
83. 81 + 20 x 0.75 9 =?
a. 122
b. 126
a. 97
b. 107
c.

130

d. 134

e. none of these

c. 87

d. 77

74. The number of boys from Anthropology 84. 22440 ? = 34 x 12


department is approximately what per cent of the
a. 55
b. 3136
total number of boys from all the departments
together:
c. 65
d. 3025
a. 15
b. 23
85. 3402 ? =
c. 31
d. 44
e. 56
a. 162
b. 21

e. none of these

e. none of these

75. What is the respective ratio of number of girls from


c. 441
d. 42
e. none of these
Philosophy department to the number of girls from
Psychology department:
86. Tanvi started a business investing Rs 45,000 .After
a. 1:2
b. 7:12
8 months Anisha joined her with a capital of Rs.
52,000.At the end of the year the total profit was
c. 5:12
d. 3:4
e. none of these
Rs. 56,165. What is the share of profits of Anisha:
a. Rs. 21,450
b. Rs. 24,440
Directions (76-78): What should come in place of the
question mark(?) in the following questions:

c.

Rs. 27,635

d. Rs. 31,765

Page | 7

e.

none of these

87. By how much is of 428 smaller than

216:
a. 61
b. 67
c.

73

d. 79

95. The profit earned after selling an article for Rs.


1,516 is the same as loss incurred after selling the
article for Rs. 1,112.What is the cost price of the
of
article:
a. Rs.1,314
b. Rs. 1,343

e. none of these

c.

Rs. 1,414

d. Rs. 1,434

e. none of these

88. One-third of a number is 96. What will 67% of that Directions (96 - 100): Each question below is followed by
two statements A and B. You are to determine whether the
number be:
data given in the statement is sufficient for answering the
a. 192.96
b. 181.44
question. You should use the data given in the statement is
sufficient for answering the question. You should use the
c. 169.92
d. 204.48
e. none of these
data and your knowledge of mathematics to choose
89. In a class of 35 students and 6 teachers, each between the possible answers:
student got sweets that are 20% of the total
number of students and each teacher got sweets Give answer (a): if the data in statement A alone are
that are 40% of the total number of students. How sufficient to answer the question, while the data in
statement B alone are not sufficient to answer the question
many sweets were there:
Give answer (b): if the data in statement B alone are
a.
245
b. 161
sufficient to answer the question, while the data in
statement A alone are not sufficient to answer the question.
c. 406
d. 84
e. none of these
Give answer (c) : If both the statement A and B together
90. Prithvi spent Rs. 89,745 on his college fees, Rs. are needed to answer the question
51,291 on personality Development Classes and Give answer (d) : If neither the statement A alone or
the remaining 27% of the total amount he had as statement B alone is sufficient to answer the question
cash with him. What was the total amount:
Give Answer (e) : If you cannot get the answer from the
a. Rs. 1,85,400
b. Rs. 1,89,600
statement A and B together, but need even more data
c.

Rs. 1,91,800

d. Rs. 1,93,200

e. none of these
91. The compound interest accrued on an amount of
Rs. 22,000 at the end of two years is Rs. 5,596.8.
What would be the simple interest accrued on the
same amount at the same rate in the same period:
a. Rs. 5,420
b. Rs. 5,360
c.

Rs. 5,280

d. Rs. 5,140

e. none of these

92. The product of two successive numbers is 4032.


Which is the greater of the two numbers:
a. 63
b. 64
c.

65

d. 66

e. none of these

93. The ages of Sonal and Nitya are in the ratio of 9:5
respectively. After 8 yrs the ratio of their ages will
be 13:9. What is the difference in yrs between their
ages:
a. 4 yr
b. 12 yr
c.

6 yr

d. 14 yr

96. Is X an even number:


A. X is divisible by 2.
B. X + an odd number is an odd number.
97. What is the three digit number:
A. One-third of that number is less by 27 of the half of
that number.
B. One-fifth of that number is 20% of that number.
98. The ages of Tanish and Shivay are in the ratio of
6:5. What is the age of Shivay:
A. The ages of Tanish and Danish are in the ratio of
3:2.
B. After 6 yrs the ratio of Danishs and Shivays ages
will be 6:7.
99. In how many days 15 men can complete a piece of
work:
A. If 6 children can complete the same piece of work
in 48 days
B. If 9 men can complete the same piece of work in
27 days.
100. What is the salary of C, in a group of A,B,C,D,E
and F whose average salary is Rs. 60,000:
A. Total of the salary of A and E is Rs. 64,500
B. Total of the salary of B and F is Rs. 52,600

e. none of these

94. 30 men can do a piece of work in 16 days. How


many men would be required to do the same work
in 20 days:
a. 12
b. 36
c. 48

d. 24

e. none of these

Page | 8

ANSWERS:SBI.M.T- 1
1.

A; G

A R

5 9 3 4
Therefore;

2.

3.

I P

I + A= I-type of conclusion.
Some pots are tubs

4 8 6 9
P A G E

All buckets are tubs

6 3 5 9
C; D N = D is sister of N. ; N K = N is mother of K.
Therefore , D is maternal aunt of K.
D N = D is sister of N. ; N + K = N is father of K.
Therefore, D is paternal aunt of K.
D x N = D is brother of N. ; N K = N is mother of K.
Therefore, D is maternal uncle of K.
B; M x T = M is brother of T. ; T + R= T is father of R.
Therefore, M is uncle of R.
R J = R is sister of J. ; J + M = J is father of M; M x T
= M is brother of T. Therefore, M is nephew of R.

All tubs are drums.


A+A= A-type of conclusion.
All buckets are drums
Conclusion III is converse of this conclusion.
Some pots are tubs.

All tubs are drums.

(4 7) : Sitting arrangement:

I +A= I-type of conclusion.


some pots are drums
Conclusion I is converse of this conclusion.
11. C; No two premises are aligned. Therefore, no
conclusion follows. Conclusions I and III form
complementary pair. Therefore, either I or III follows.
12. A; Some trucks are boats

No boat is jeep.
I + E = O-type of conclusion.
4.

5. D

6. A

7. C

(8-12):

Some trucks are not jeeps.


(13-17):

(I)
(II)
(III)
(IV)

All pins are rods Universal affirmative (Atype)


Some rods are chains particular affirmative
(I-type)
No pin is hammer universal negative (Etype)
Some pins are not hammers particular
negative (O-type)
Some rods are chains

P Q means P Q
P % Q means P Q
P @ Q means P = Q
P $ Q means P < Q
P Q means P > Q

13. D; H T = H T ; T % M = T M ; M F = M > F
Therefore ; H T M > F
Conclusions:
All chains are hammers
I.
F $ T = F < T : not true
I + A= I-type of conclusion.
II.
H M = H > M : not true
Some rods are hammers
14. C ; B @ N B = N ; N T = N T ; T $ K = T <K
Conclusion II is converse of this conclusion.
Therefore; B = N T < K
Conclusions:
Conclusions I and III form complementary Pair.
I.
T @ B T = B: not true
Therefore, Therefore ,either I or III follows
II.
T $ B = T < B: not true
9. A; All the three premises are particular affirmative.
Either I or II is true
Therefore, no conclusion follows.
10. B; Some pots are buckets
15. D; R $ J = R < J ; J F = J > F ; F % H = F H
8.

D;

All buckets are tubs.

Therefore, R < J >F H


Conclusions:
I.
H J = H > J: not true

Page | 9

II.
R $ F : R < F : not true
16. B; J D = J > D ; D @ N D= N ; N % F = N F
Therefore, J > D = N F
Conclusions:
I.
J F = J > F: not true
II.
F D = F D: true
17. A; B T= B>T ; T $ H = T < H ; H @ M H =M
Therefore; B > T < H = M
Conclusions:
I.
M T = M > T : true
II.
B H = B> H: not true
18. E;
19. E; The code for economics is either ka or ha
20. B; supply = re ; related = ab ; market = ni
21. D ; more = ja
22. B; market = ni, needs = de ; more = ja
The code for customers may be pa.
23. E ; needs = de
(24- 28) : After careful analysis of the given input and
various steps of rearrangement, it is evident that in each
step one number and one word are rearranged, it is evident
that in each step one number and one word are rearranged.
In the first step the highest number moves to the extreme
left position while the word which comes first in the
alphabetical order moves to the extreme right position. In
the next step the second highest number moves to the
second position from the left and the word which comes
second in the alphabetical order moves to second position
from the right. The same procedure is continued till all the
numbers get arranged in descending order from the left and
the words get arranged in alphabetical order from the right.

29. C

31. D

32. B

33. E

Input: hard work pays 96 42 in 79 long run 18 25 57


Step I: 96 work pays 42 in 79 long run 18 25 57 hard

30. A

36. C

Step II: 96 79 work pays 42 long run 18 25 57 in hard


Step III: 96 79 57 work pays 42 run 18 25 long in hard

45. D

Step IV: 96 79 57 42 work run 18 25 pays long in hard

50. C

Step V: 96 79 57 42 25 work 18 run pays long in hard

53. D

Step VI: 96 79 57 42 25 18 work run pays long in hard

56. B

34. B

35. D

37. A

38. D

39. B

40. B

41. B

42. C

43. B

44. D

46. A

47. C

48. B

49. E

51. E

52. A

54. C

55. E

57. C

58. D

59. A

60. B

62. C
63. D
64. A
65. E
61. E
24. C
25. A; There are six elements (pays, 42, run, 18, 25, long) 66. A; The corresponding angle for commuting = 79.2o
between work and in in the step III
45800
o
o
360 = 45800 then 79.2 =
79.2 = . 10076
26. D; The word pays is fourth from the left in the step II.
360
27. B; The word run is seventh from the right in the step 67. D; Required ratio = 39.6 : 82.8 = 396 : 828 = 11:23
68. B; Corresponding angle for entertainment and
IV.
o
shopping together = 36 + 68.4 = 104.4
28. C;
45800
o
104.4 =
104.4 = 13282
(29 35):
360
69. E ; corresponding total angle for groceries ,
entertainment and investments together = 82.8 + 36 +
54 = 172.8
Corresponding angle for commuting = 79.2
172 .8
Required percentage =
100 218
79.2
39.6

70. C; Required percentage =


100 = 57.89
68.4
71. E ; Number of girls in all departments = 140 + 300 +
180+250 + 240 = 1110
300
Required percentage =
100 27
1110

Page | 10

72. D; Total number of boys in all the departments = 80 +


According to the question, 73% of x = (89745 +
73
100
200 + 100 + 150 + 120 = 650
51291) =
= 141036 = 141036
=
100
73
Number of girls = 1110
. 193200
Required difference = 1110 650 = 460
91. C ; Let the rate of interest be R% per annum.
650
73. C; Average number of boys =
= 130

5
= +
. = +

74. A; Number of all boys = 650


.=++
Number of boys from Anthropology department =
=+
.
100
100

Required percentage =
100 = 15.38 15
+
=
=
+
=
=
650

75. B; Number of girls from Philosophy department = 140

=
Number of girls from psychology department = 240

required ratio = 140:240 = 7:12



Simple
interest
=
=

76. B; ? =

= .

77. D; ? =
78. A; ? = (. ) () = ; Approximate 92. B;
93. E; Let the present ages of Sonal and Nitya be 9x and
answer = 3000
5x yrs respectively.
79. C; The given number series is based on the following
9+8
13
According to the question;
=
81 + 72 =
Pattern:
5+8
9
2
2
2
2
32
20 + 2 = 24 ; 24+3 =33 ; 33 + 4 = 49 ; 49+ 5 = 74 ;
65 + 104 = = 2 ; Required difference = 9x
2
2
16
74 + 6 = 110 ; ? = 110 + 7 =110 +49 = 159
5x = 4x = 4x2 =8 yrs
80. D; The given number series is based on the following
94. D ;
Pattern:
16 x 1.5 = 24 ; 24 x 2 = 48 ; 48 x 2.5 = 120 ; 120 x 3 =
360 ; 360 x 3.5 = 1260 ; ? = 1260 x 4 = 5040
81. A; The given number series is based on the following
Pattern:
8 x 4 -1 = 31 ; 31 x 4 -2 = 122; 122 x 4 3 = 485 ; 485
20 : 16 = 30 : x 20 x x = 16 x 30 x = 24
x 4 4 = 1936 ; 1936 x 4 5 = 7739 ; ? = 7739 x 4 - 6
= 30950
95. A; Let the cost price of the article be Rs. x.
82. B; The given number series is based on the following
According to the question, 1416 x = x 1112
Pattern:
2x = 2628 x = 2628/2 = Rs. 1314
499 + 1 x 123 = 622 ; 622 + 2 x 123 = 868 ; 868 + 3 x 96. D ; From statement A, A number divisible by 2 is an
123 = 1237 ; 1237 + 4 x 123 = 1729 ; 1729 + 5 x 123 =
even number.
2344 ; ? = 2344 + 6 x 123 = 3082
From statement B; 2 + 3= 5 (odd number)
83. C; ? = + . = + =
4 + 9 = 13 (odd number)
22440
2240
84. D;
= 34 12 ? =
= 55 ? = 55 55 = 97. A; From statement A, Let the number be x.
?
34 12

32
= 27
= 27 = 27 6 = 162
3025
2
3
6

Statement B is superfluous.
85. B ;
= = ? =
=
?

98. C; From both statements, T : S = 6:5 ; T: D = 3:2 or


86. E; Ratio of the earned profit = Ratio of the equivalent
6:4 then T:S:D=6:5:4
capitals of Tanvi and Anisha for 1 month = 45000 x 12
Let the present ages of Danish and Shivay be 4x and
: 52000 x 4 = 135 : 52
5x years respectively.
Sum of ratios = 135 + 52 = 187
4 + 6 6
52
= =3
Anishas share = .
56165 = . 15618
5 + 6 7
187
5
1
Hence, the age of Shivay can be determined.
87. C; Required difference = 216 428 =
6
4
99. B; from statement B,
180 107 = 73
88. A; let the number be x. According to the question,

= 96 = 96 3 = 288
3

67% of 288 =

288 67
100

= 192.96

15:9 = 27:x 15 x x = 9 x 27 x = 81/5 days


100. E; From statement A, A +E = 64500
From Statement B, B + F = 52600
Number of sweets got by each teacher =
= 14
100
C+D = (A+B+C+D+E+F)- (A+B+E+F) , hence ,
Total number of sweets = 35 x 7 + 6 x 14 = 245 +
the age of C cannot be determined.
84 = 329

89. E ; Number of sweets got by each student =

35 20

100
35 40

=7

90. D; Let the total amount with Prithvi be Rs x.


Percentage of amount spent = (100 27) = 73%.

Page | 11

AAA

BRIGHT ACADEMYTM

CENTRE FOR EXCELLENCE IN EDUCATION

SBI MOCK TEST-2


REASONING

3.

Which of the following would come in the place of


the question mark based upon the given seating

Directions (1-5): Study the following information carefully


and answer the given questions:

arrangement: VS XR TV RP ?
a. SW

b. WX

c. QW

d. QX

P,Q,R,S,T,V,W and X are captains of eight different cricket


teams namely Australia, New Zealand, India, Pakistan, Sri

e. VR

Lanka, England, West Indies and South Africa, but not 4.

Which of the following is true with respect to the

necessarily in the same order. All of them are seated

given seating arrangement:

around a circular table and are facing the centre.

a) R is the captain of South Africa

P is sitting third to the left of the Sri Lankan Captain. Only

b) W is an immediate neighbour of V.

two people sit between T and W. Neither T nor W is an

c) Captains of Australia and England are immediate

immediate neighbour of P. Neither T nor W is the captain of

neighbours of each other.

Sri Lanka. The captain of South Africa sits second to the

d) Four people sit between W and Q

right of S. S is not an immediate neighbour of P. S is not Sri

e) X sits second to left of S.

lankan Captain and P is not the captain of South Africa. The 5.

Who among the following is the Indian Captain:

Australian captain sits third to left of V. Australian and Sri

a. Q

b. V

Lankan captains are not immediate neighbours of each

c. X

d. T

e. cant determine

other. Only one person sits between S and Indian Captain. Directions (6-7) : study the following information carefully
Captains of Pakistan and New Zealand are immediate and answer the given questions:
neighbours of each other. S is not the captain of New Amongst five friends, each got a different percentage of
Zealands team. Only one person sits between Q and the marks in the examination. Poonam scored more than Ben
captain of England. The captain of England is immediate but less than Ajay. Ajay scored 70% marks. Shreya scored
neighbour of X. W and Q are not immediate neighbours of less marks only than Kim. The one who scored the
each other.
minimum marks scored 65% marks and the one who
1.

2.

How many people sit between T and the captain of scored the highest, scored 87% marks.
England when counted in clockwise direction from 6.
T:

Who amongst the following scored the second

a. None

b. one

a. Ben

b. Kim

c. two

d. four

c. Shreya

d. Ajay

e. five

lowest marks:

e. Poonam

Who amongst the following is the captain of the 7.


Australian team:

Who amongst the following is the most likely to

a. P

b. V

a. Ben

b. Poonam

c. W

d. T

c. Shreya

d. Kim

e. Q

have scored 82% marks:

e. Either Kim or Ben

Page | 1

8.

Which of the following symbols should replace the

II.

No building is an apartment

question mark in the given expression in order to 12. Conclusions:


make the expression K H as well as M > J

I.

All buildings being flats is a possibility

definitely true:

II.

All apartments being buildings is a possibility

HI=J?KL<M
b.

a. >
9.

c.

(13-14) : Statements:
d. < or

e. =

Some oceans are seas.

In which of the following expressions will the

All oceans are rivers

expression P > S be definitely not true:


a. P > Q R = S

b.S R Q < P

c. R = P > Q S

d.S > Q R < P

e. S < Q < R < P


10. Which of the following symbols should be placed
in the spaces respectively (in the same order from
left to right) in order to complete the given
expression in such a manner that N < K definitely
holds true:

No river is a canal
13. Conclusions:
I.

All rivers can never be oceans

II.

All canals being oceans is a possibility

14. Conclusions:
I.

No ocean is a canal.

II.

At least some seas are rivers

15. Statements:
No day is night.

K __ L __ M __ N

All nights are noon.

a. , = , >

b. , < ,=

c. , = , <

d. > , , <

No noon is an evening.
e. none

Directions (11-15) : In each of the questions below are


given two/three statements followed by two conclusions
numbered I,II. You have to take the given statements to be
true even if they seem to be at variance from commonly
known facts. Read all the conclusions and then decide
which of them given conclusions logically follows from the
given statements disregarding commonly known facts:
Give answer (a) if only conclusion I follows.
Give answer (b) if only conclusion II follows

Conclusions:
I.

No day is noon.

II.

No day is an evening.

Directions (16-20): Study the following information to


answer the given questions:
Ten people are sitting in two parallel rows containing five
people each, in such a way that there is an equal distance
between adjacent persons. In row- 1

- P,Q,R,S ,T are

seated and all of them are facing south. In row-2


A,B,C,D,E are seated and all of them are facing North.

Give answer (c) if either conclusion I nor conclusion II follow Therefore, in the given seating arrangement each member
Give answer (d) if neither conclusion I nor conclusion II seated in a row faces another member of the other row.
follow

D sits third to the left of A. P faces immediate neighbour of

Give answer (e) if both conclusions I and II follow.

D. R sits second to the right of P. Only one person sits

(11-12): Statements:

between Q and S. B and E are immediate neighbours of

All buildings are houses


No house is an apartment
All apartments are flats
11. Conclusions:
I.

each other. E does not face P and Q.


16. How many persons are seated between Q and T:
a. None

b. one

c. two

d. three

e. cant determine

No flat is a house

Page | 2

17. Four of the following five are alike in a certain way a particular rule in each step. The following is an illustration
and thus form a group. Which is the one that does of an input and rearrangement.
not belong to that group:

Input: sum 28 have 19 96 48 luck nice 78 rope

a. R

b.S

Step I: have sum 28 19 48 luck nice 78 rope 96

c. C

d. T

e. A

Step II: luck have sum 28 19 48 nice rope 96 78

18. Who amongst the following represent the people Step III: Nice luck have sum 28 19 rope 96 78 48
sitting exactly in the middle of the rows:
Step IV: rope nice luck have sum19 96 78 48 28
a. P,E
b. S,D
Step V: sum rope nice luck have 96 78 48 28 19
c. S,A
d. A,R
e. P,B
And step V is the last step of the rearrangement
19. Which of the following is true regarding B:
As per the rules followed in the above steps, find out in
a. A and C are immediate neighbours of B
each of the following questions the appropriate step for the
b. B sits at one of the extreme ends of the line
given input
c. Q faces B
Input: 49 last zen 16 82 yet can vast 33 aim 87 54
d. T is an immediate neighbour of the person facing
23. How many steps would be needed to complete the
B
e. D sits to the immediate left of B.

arrangement:

20. Four of the following five are alike in a certain way

a. IV

b. V

and thus form a group. Which is the one that does

c. VI

d. VII

not belong to that group:


a. T-E

b. Q-C

c. S-B

d. R-A

e. None

24. Which step number would be the following output:


Vast last can aim zen 16 yet 33 87 82 54 49
e. P-D

a. III

b. II

Directions (21-22): Study the following information to

c. VII

d. IV

answer the given questions:

e. there will be no such step

In a certain code her idea has merit is written as fo la bu 25. Which of the following would be the step I:
na , merit list has been displayed is written as jo ke la si
a. Aim 49 can zen 16 yet vast 33 54 87 82
na and her name displayed there is written as ya si u zo,
b. Vast last can aim zen 16 yet 33 87 82 54 49
name in merit list is written as na ya go ke.

c.

21. What does ke stand for:

d. Aim 49 last zen 82 yet can vast 33 87 54 16

a. Been

b. has

c. merit

d. name

Zen 49 last 16 82 yet can vast 33 aim 54 87

e. None of these
e. list

Directions (26-30): Study the following information to

22. Which of the following represents name has been answer the given questions.
displayed:

Eight friends Meenal, Rumia,Shikha, Ali, Peter, Harleen,

a. Ya la ke si

b. jo si ya la

Ketan and Bharat are sitting around a square table in such

c. si jo ke na

d. buy a ke la

a way that four of them sit at four corners of the square

e. ya si jo zo
Directions (23-25) : Study the following information to
answer the given questions:
A word and number arrangement machine when given an
input line of words and numbers rearranges them following

while four sit in the middle of each of the four sides. The
ones who sit at the four corners face the centre while those
who sit in the middle of the sides face outside.
Bharat sits second to the right of Shikha. Bharat does not
sit at any of the corners. Meenal sits third to the right of

Page | 3

Peter. Peter is not an immediate neighbour of Shikha.

Which of the following statements would weaken the

Rumia and Ketan are immediate neighbours of each other

above

but Rumia does not sit at any of the corners of the table.

belonging to western parts of India should move to

Harleen is neither an immediate neighbour of Peter nor

south India to increase their life expectancy?

Shikha.

a. The average life expectancy of population living in

mentioned

studys

proposal

that

people

Eastern part of the country is also less than the

26. Four of the following five are alike in a certain way

population living in south India.

and so form a group. Which is the one that does

b. Nearly 80% of the population in southern India has

not belong to that group:


a. Peter

b. Rumia

c. Harleen

d. Shikha

a minimum age of 83 years.


e. Bharat

b. Rumia

c. Shikha

d. Peter

Higher life expectancy in southern India can be


ascribed to the genetic makeup of the population

27. Who sits third to the left of Ali:


a. Bharat

c.

belonging to that area.


d. The average life expectancy of south India is
e. Cant determine

28. What is the position of Peter with respect to


Meenal:

comparable to the best averages in the world.


e. Higher life expectancy in southern India can be
attributed to better environmental conditions and

a. To immediate left

b. second to the left

c. third to the left

d. third to the right

better healthcare facilities.


Directions (32-35): Read the following information carefully
and answer the questions which follow:

e. second to the right

29. Who amongst the following sits second to the Fortunately, more and more countries are shifting their
focus away from industrial development to control of climate

right of Ketan:

change these days.

a. Shikha

b. Ali

c. Bharat

d. Harleen

30. Who

amongst

the

A. The countries which focus more on controlling

e. Meenal

following

represent

the

climate change than industrial development are

immediate neighbours of Harleen:

only richer ones which can afford to concentrate on

a. Meenal, Ketan

b. Bharat,Rumia

areas other than industrial development.

c. Bharat, Meenal

d. Ali, Rumia

B. Many countries had once prioritized

industrial

development which proved to be harmful to the


e. Ali, Ketan

environment in the longer run.

31. Read the following information and five statements

C. Some experts are of the view that climate is not as

given below it carefully and answer the question

alarming an issue as it is made to be because it is a

which follows.

natural phenomenon and has been occurring

Except

from

research

report-

Average

life

regularly throughout the history of earth.

expectancy in Southern part of India is far more than

D. If climate change continues at the present rate, It

that in Western India. While the average life of a native

would bring in large scale destruction to human

of south India is 82 years, the average life of a native

habitation in a very short time.

of western India is only 74 years.

E. Industrial development is one of the biggest but

Based on the above fact, the proposal that the above

definitely not only reason behind global warming.


study makes is that if an individual moves from 32. Which of the statements numbed A,B,C,D,E
western India to south India, his/her life expectancy
mentioned above would weaken the argument for
would immediately increase by eight years.

Page | 4

the

need

of

shift

away

from

industrial

ENGLISH LANGUAGE

development to that of controlling climate change:


a. A

b. B

c. C

d. D

Directions (36-45): Read the following passage carefully


e. both B and D

and answer the questions given below it. Certain words/


phrases have been printed in bold to help you locate them

33. Which of the statements numbered A,B,C,D and E


can

be

an

information

assumed/inferred
given

in

the

from

the

while answering some of the questions:

fact/

statement?(an

assumption is something supposed or taken for


granted and an inference is something which is
not directly stated but can be inferred from the

The prices of gold has gone up from $ 256 an ounce in


2001 to $ 1,424. Meanwhile price levels have struggled or
crashed with respect to almost all other asset classes.
Central banks have slashed interest rates. Yet gold prices,
it has been predicted, may go up and up. The many

given facts):
a. E

b. C

c. A

d. B

reasons for this renewed love are convincing. Interestingly,


not long ago pundits had predicted the end of gold as the
e. Either D or A

worlds default asset class and were clubbing it with


34. Which of the following can be inferred from commodities. It appears that the yellow metal is making a
statement (E) if it is considered to be true with comeback to reassert the pre-eminence it has enjoyed for
regard to the given information:

5,000 years of history.

a. Nations also need to focus on sources other than


those generated due to industrial development.
b. Other sources of pollution have more adverse
effects as compared to those generated due to
industrial development.
c.

Its supply is falling. No new mines have been discovered.


The existing ones are getting exhausted and miners are
digging as deep as 5 km. Gold content in ore has come
down from almost 12 gm a tone to 2 gm. And it costs more
and more to take that out. Environmental concerns have

Unlike older times, industrial development has


ceased to be a reason behind global warming
these days.

also contributed to mine-owners problems. The wages of


miners are going up, so is the cost of providing them safety
and security.

d. If industrial development stops, global warming


Emerging economics such as China and India are

would automatically come to an end.


e. If sources other than industrial development are
identified and controlled, global warming will end

of

the

the dollar. While the U.S. has a reserve of 9,200 tonnes of


gold. China has 1,054 tonnes and India 565 tonnes. No

completely.
35. Which

accumulating gold in order to reduce their dependence on

statements

numbed

A,B,C,D,E

mentioned above represents a reason behind


curtailing industrial development by some of the

wonder , as emerging economic superpowers China and


India want to add to the reserves. Industrial use of gold is
on the rise the world over. With the US economy still drifting
with the threat of the dollar losing its undisputed position of

countries:
a. Either C or E

b. D

c. C

d. Either A or C

reserve currency the rush to gold is increasing.


e. A

Added to all this is the rekindled investor-preference of


gold. Money is moving away from mutual funds and equities
and the once fashionable and often discredited hedge funds
are also getting into gold, Exchange traded funds (ETFs)
are channeling ever more funds to gold. Some pension
funds are increasing the proportion of gold in their basket of

Page | 5

assets. Given all this, gold can go nowhere but up. That is evaporate at the click of a key. Of course, governments
the consensus.

such as China and India are also betting on gold and

Everyone seems to be joining the new gold rush. But is increasing their reserves. But then,
governments can make no mistakes?
everything well with gold? Or is it a bubble building up?

whoever

said

Consider the conventional wisdom. Money generally gets The intrinsic value of gold has not gone up from $ 255 to
distributed, though not in any fixed proportion, among 1,424 in 10 years, Gold is the most recycled commodity. Of
assets such as real estate, stocks, cash, government the annual production of 2,500 tonnes, about 50 per cent
securities, gold, commodities, and in new investments in goes to make jewellery and it is almost entirely recycled.
factories and machinery. There is no state of equilibrium in The rest goes to industrial and other uses, and even here
a global economy. Money gets transferred across the recycling rate is high. In other words, all that demand is
geographical boundaries and asset classes based on artificial and can be deflated in no time. There is no need to
anticipated gains. As long as the flow is reasonable and is have excess supply to lead to a price crash, unlike other
generally in line with the increase in returns, this works well. products. The sheer fact that gold is only a hedge
But when everyone rushes to the same destination, we instrument and does not serve any practical use by itself,
are looking for trouble. Excess demand, though often will negate the there-is-no-new-supply theory. Someone
artificial, creates excess supply, as in the case of real somewhere is watching for the perfect moment to disgorge
estate. Excess supply leads to price crashes.
Is something similar happening in gold? The general

the hoard, to create sudden panic and buy up following a


crash.

consensus is no. Gold is different. It has never let anyone There is no sign that a crash is going to come tomorrow, or
down in 5,000 years. It is indestructible. Its supply is limited. for that matter next year or the year after, It may still go up
The argument in the case of gold is that excess demand for two or five or even 10 years. But crash it will, if we are to
cannot create excess supply as the total world supply is go by the economic history of boom and bust. Also, the
limited. But this time it is different .Is it really so? Gold has higher it goes and the longer it stays there, the more painful
also gone up and down in the past. It was $ 424 announces the crash is going to be, especially for India, Indians sit on
in 1990 before crashing to $ 255 in 2001. Still, it moves only an estimated 18,000 tonnes. India has always has the
within a range and huge fluctuations are not possible in largest gold reserve with individuals. Imagine what will
gold , argue some people, Actually, gold gave much returns happen to millions of Indians if gold were to crash. A crash
in the 1980s, only to stagnate and lose those gains in the of gold will be the crash of the Indian Economy.
1990s.

That should make us more responsible. That makes it

A crash of gold prices could be the ultimate crash, nothing imperative for our economists to track gold movement. That
like we have seen. No one has managed to discredit the makes it important for our financial wizards to prevent a
yellow metal in 5,000 years. But it appears that for the first bubble in gold. That should force us to act before it
time in history the ETFs, the hedge funds and the happens. Never in history have we had so much idle money
governments are about to do the undoable. The fact that it chasing so little gold. Gold is losing its respect as the
has not already happened is no guarantee that it will not default and fail-safe as-set class and becoming a
happen. Look at all the easy money coming into gold. All speculative instrument. This shift of gold from being an item
those who have shifted money from real estate, mutual of passive wealth to an instrument of speculation is
funds, pension funds, hedge funds and stocks are pouring it dangerous. Gold is being talked up by crafty. Speculators
into gold. Gold ETFs are the fastest growing investment and unsuspecting governments. And these predictions are
vehicles today . This is all real quick money, but can being made by those sitting on gold worth billions of dollars
bought at yesterdays prices.

Page | 6

36. Which of the following best explains When

d. It will become a commodity so highly-priced that

everyone rushes to the same destination in the


context of the passage:

e. None of these

a. Only real estate is invested in by a majority.


b. A majority invests in everything else except for

c.

40. Which of the following , according to the passage,


is/are the reason/s for the increase in gold prices:

gold.

I.

Failure in discovering new mines.

Everyone wants to become rich at the same time

II.

Depleting content of gold in ores

III.

Increase in remuneration of miners.

d. Everyone rushes to the same place in order to buy


gold.
e. Nothing else but gold is invested in by a majority.
37. Which of the following is/are the general opinions
about gold:
I.

no Indian will be able to buy it.

Its supply, although limited, will be able to


meet its demand.

II.

The price of gold will keep on increasing

III.

It is the best investment at present.

a. Only I and III

b. only I and II

c. Only I

d. only II and III

a. Only II

b. only I and III

c. only III

d. only II and III

e. all I,II,III
41. Why, according to the author, is the demand for
gold artificial:
a. Most gold that is used is also recycled and there is
no need for fresh supply.
b. Industries demand gold for the purpose of stocking
and do not actually use it for any process.
c.

e. all I,II,III

The supply of gold is too little as compared to its


demand thereby making it artificial.

38. Which of the following is true, in the context of the

d. The amount of gold actually needed by industries

passage:

and others is much lower and the amount quoted

I.

has been hyped by the government.

Gold is being increasgly used for industrial


purposes the world over.

II.

III.

India has lesser gold reserve as compared to Directions (47-48): Choose the word/group of words which
US and China
is most similar in meaning to the word/group of words
Countries like US and china have now printed in bold as used in the passage:
stopped investing in gold for fear of its prices
crashing.

42. Concerns
a. Relations

a. Only I and III

b. only I and II

c. only I

d. only II and III

e. all A,B,C
39. What is the authors fear with regard to gold:
a. Its limited supply will affect the operations of most
industries in India.
b. Its surging demand will not be met with an
adequate supply and will bring about a down turn
in the Indian economy.
c.

e. None of these

b. distracters

c. apprehensions

d. organizations

e. institutes
43. Consumed
a. Eaten

b. used

c. cleared-up

d. put-away

e. devoured

Directions (49-50): Choose the word/group of words which


is most opposite in meaning to the word/group of words
printed in bold as used in the passage

Its prices will suddenly dip which in turn will 44. Exhausted
adversely affect many people in India.

a.

Started

c. fresh

b. revitalize
d. replenished

Page | 7

e. tired

c. C

45. Artificial
a. Genuine

d. E

e. A

Directions (49-58): In the following passage there are


b. simulated

c. unadulterated d. false

blanks , each of which has been numbered. These numbers


e. valid

Directions (46-48): Rearrange the following six sentences


(A), (B),(C),(D) ,(E) and (F) in the proper sequence to form

are printed below the passage and against each, five words
are suggested, one of which fits the blanks appropriately.
Find out the appropriate word in each case:

a meaningful paragraph, then answer the questions which Can an experiment conceived, carried out, and reported in
kids- speak with pencil-coloured figures and hand-written
follow:
A. Ironically the same parents who are considered to
be ignorant are thought to very enlightened in
choosing private schools over-state run ones.
B. This is all the more reason why we should include
them during the planning and implementation of the

tables by school children aged 8 to 10 years get published


in a highly rated international journal following a peerreviewing process? Twenty-seven school children from a
primary school in UK have proved this is 49 if a simple but
novel scientific question raised is 50 in a scientific way.
Their paper was published in the Royal Societys Biology

system.
C. This is apparent at every stage from policy making
to implementation as critical decisions are made
without the participation of the stakeholders, an
attitude that can only be described as either

D. In reality, every parent decides which school is a


good one based on his/her own set of values,

E. The root cause of most of the ills that plague our


education system is the enormous distance that
separates the power centres within the system and
the schools where the action takes place.
F. It is often said in defense of such an approach that
poor parents are too ignorant to be partners in a

46. Which of the following sentence should be the last


(sixth) sentence after rearrangement:

d. D

e. E

Third sentence after rearrangement:


b. B

c. C

d. D

that schoolchildren guided by gifted teachers can think and


52 out experiments like any hard-wired scientists. For these
Science also become cool and fun. This refreshing
approach turns the spotlight on the best methods of
teaching science. The 54 learning system adopted by most
schools in India, even classroom study combined with some
laboratory work with predefined outcomes , does very little
to 55 curiosity and interest in science. Is that one of the 56

science rarely comes out of Indian laboratories? The


children at the UK school had their gifted teacher to guide
them. Scientists from Indias space and atomic energy

47. Which of the following sentence should be the

a. A

understating of how bees perceive coloured patterns and

why out of the box thinking that produces path-breaking

meaningful dialogue.

c. F

which colour of flower to forage from . Considering that our

kids , doing science changed their 53 of the subject.

perceptions and aspirations.

b. B

Combination of colour and spatial relationships in deciding

scenes is inadequate, this inspiring outcome has shown

arrogance or indifference.

a. A

Letters journal. Their 51 was that bumblebees can use a

departments and in some other places where serious


science is done can take a 57 out of the schools book and
58 the way in engaging with school pupils and getting them
to do real science.
49. a. done

e. F

c. potential
48. Which of the following sentence should be the 50. a. questioned
second sentence after rearrangement:
c. retorted
a. F

b. D

51. a. question

b. unlikely
d. promising

e. possible

b. said
d. answered

e. address

b. finding

Page | 8

a. methodology
52. a. wage
c. execute
53. a. option
b. visual
54. a. revolutionary

d. result

e. studies

b. create
d. carry
b. lives
d. demands

b. simulate

57. a. thread
c. example
58. a. lead
c. deliver

e. perception

b. radical

55. a. stimulate

c. reasons

it. The error if any will be in one part of the sentence, the
number of that part will be the answer. If there is no error,

d. adequate

56. a. cause

whether there is any grammatical error or idiomatic error in


e. attempt

c. rote

c. make

Directions (62-65) : Read each sentence to find out

d. peek

the answer is (e). No error. (Ignore the errors of


punctuation, if any)

e. bore

62. The biggest (a)/ health burden that India (b)/ is set
to face in the coming years (c)/ is tackle cancer.

e. judge

b. root

(d)/ no error (e).


63. Illegal land mining in the peripheral areas (a)/ of

d. issues

e. sources

b. leaf

the district continues to remain a big problem for


(b)/ forest range officers as yet another instance

d. look

e. pages

b. start

illegal (c)/ mining was reported yesterday.(d)/ no


error (e)

d. paved

e. ahead

64. The policemen, who (a)/ was deployed heavily (b)/

Directions (59-61): Each question below has two blanks,

in the area, did nothing to (c)/ dissuade the

each blank indicating that something has been omitted.

protesters. (d)/ no error (e)

Choose the set of words for each blank which best fits the 65. Festivals are prime occasions (a)/ for splurging on
meaning of the sentence as a whole.

presents

59. The sun has constantly been ____ and reappearing

situation , (b)/ the youths is gungho (c)/ about

and that has got the temperature ____ over the


past two days.
a. Vanishing, control.

b. hiding, fluctuating

c. appearing, soaring

d. going, unpredictable

and

owing

to

improved

economic

breaking all previous records.(d)/ no error (e)

QUANTITATIVE APTITUDE
66. Kamya purchased an item of Rs. 46,000 and sold it
at a loss of 12 percent. With that amount she

e. disappearing, unusual

purchased another item and sold it a gain of 12


60. The teacher called upon the students to ____
interest in science and participate in various

per cent. What was her overall gain/loss :


a. Loss of Rs. 662.40

b. Profit of Rs. 662.40

c. Loss of Rs 642. 80

d. profit of Rs. 642. 80

orientation programmes in order to ____ their


scientific skills.
a. Take, learn

b. place, further

c. develop, hone

d. generate, mix

e. none of these
67. The sum of six consecutive even numbers of set A
is 402. What is the sum of another set B of four

e. envision, share

consecutive numbers whose lowest number is 15


61. The police commissioner may ____ the state
government to close five police station ____ of

less than double the lowest number of set-A:


a. 444

b. 442

c. 440

d. 446

poor infrastructure and bad location.


a. Order, in the light

b. request, because

c. ask, reason

d. plead, due

e. none

68. Call rate of a sim company-A is 1 paisa for every 3


seconds. Another sim company B charges 45

e. command, in lieu

paise per minute. A man talked 591 seconds from


sim company A and 780 seconds from sim

Page | 9

company B. What would be the total amount he

Boys

spent:
a. Rs. 7.80

b. Rs. 7.40

c. Rs. 7.46

d. Rs. 7.82

School F
e. Rs. 8.46

School E

69. A 280 metre long train moving with an average

School D

speed of 108 km/hr crosses a platform in 12

School C

seconds. A man crosses the same platform in 10


seconds. What is the speed of the man in
metre/second:
b. 8 m/s

c. 12 m/s

d. cant determine

ratio

between

the

three

angles

of

the fourth angle of the quadrilateral is 36 . What is


the difference between the largest and the second
smallest angles of the quadrilateral:
o

c. 72

b. 108
d. 96

200

400

600

800 1000 1200 1400

e. none

quadrilateral is 13:9:5 respectively. The value of

a. 104

School A
0

a. 5 m/s

70. The

School B

71. What is the total number of girls in School C,


number of girls in school E and the number of
boys in school D together:
a. 1700

b. 1900

c. 1600

d. 1800

e. none

72. What is the respective ratio between the number of

e. none

Directions (71- 75) : study the following pie-chart and bar


chart and answer the following questions.
Percentagewise Distribution of students in six different

boys in school-C, number of girls in school-B and


total number of students in school E:
a. 45:7:97

b. 43:9:97

c. 45:7:87

d. 43:9:87

e. none

73. What is the difference between the total number of

schools
Total number of students = 6000
Percentage of students

students in school F and the number of boys in


school E:
a. 820

b. 860

c. 880

d. 900

e. none

74. In which school the total number of students (both


boys and girls) together are equal to the number of
girls in school-E:

Number of boys out of 6000 students in each school


separately

a. A

b. B

c. C

d. D

e. F

75. Number of girls in School A is approximately what


percent of total number of students in School B:
a. 55

b. 50

c. 35

d. 45

e. 41

Directions (76-80): Study the table carefully to answer the


questions that follow:

Page | 10

Number of athletes (in hundreds) who participated in a In a ship there are 1200 passengers. 18 % of the total
sports event from five different countries over the number of passengers is from Britain. Two-fifth of the total
years

number of passengers is from South Africa. 6% of the total


number of passengers is from Madagascar. Remaining
number of passengers is from India. 25 % of the number of
passengers from Britain is female. Half the number of
passengers from South Africa are male. There is no female
passenger from Madagascar. Two-third of the number of
passengers from India is females.
81. What is the respective ratio between the number of

76. In which year was the total number of participants


(athletes) second highest from country C:
a. 2005

b. 2006

c. 2007

d. 2008

passengers from Madagascar, number of female


passengers from South Africa and the total
number of passengers from India:

e. none

77. Which was the average number of female athletes


who participated from Country B over all the years
together:

a. 2:5:18

b. 3:10:18

c. 3:11:18

d. 2:18:5

e. none

82. Number of male passengers from South Africa is


approximately what percentage of the total number

a. 1200

b. 400

c. 600

d. 1800

of passengers from Britain:


e. 3600

78. What was the approximate percentage decrease in


the number of male athletes who participated from
country C in the year 2007 as compared to the
previous year:
a. 21

b. 30

c. 35

d. 39

a. 111

b. 115

c. 120

d. 125

e. 131

83. What is the average number of male passengers


from all the four countries:
a. 154.5

b. 164.5

c. 145

d. 164

e. none

e. 25
84. What is the difference between the number of male

79. Number of female athletes who participated from


country E in the year 2009 was approximately what
percentage of the total number of athletes who

passengers from Madagascar and the number of


male passengers from India together :
a. 64

b. 82

c. 74

d. 72

participated from country-B in the year 2008:


a. 40

b. 46

c. 50

d. 56

e. 60

e. none

85. What is the total number of male passengers from


Britain and female passengers from India together:

80. From which country is the difference between the

a. 340

b. 420

c. 350

d. 460

number of male and female participants second


highest in the year 2006:
a. A

b. B

c. C

d. D

Directions (86 87): What will come in place of the


e. E

Directions (81 85) : Study the information carefully to


answer the questions that follow:

e. none

question mark (?) in the following questions:


86. = ( ? )
a. -256

b. 256

c. 12

d. 144

e. -16

Page | 11

87.

II. + =

() + =?
a. 42

b. 1024

c. 1764

d. (1024)2

96. The ratio between the speed of a bus and train is


e. 32

Directions (88 89): What approximate value will come in


place of the question mark(?) in the following questions
(you are not expected to calculate the exact value):
88. 89.988% of 699.9 + 50.002% of 999.99 170.015 = ?

89.

a. 990

b. 900

c. 920

d. 960

three-fourth of the speed of the car. How distance


will the train cover in 7 hours:
a. 760 km

b. 756 km

c. 740 km

d. cant determine

e. none

97. Ramans present age is three times his daughters


present age and nine-thirteenth of his mothers
present age. The sum of the present ages of all of

a. 760

b. 800

them is 125 years. What is the difference between

c. 690

d. 870

of 720 km. in 9 hours. The speed of the bus is

=?

e. 860

15:27 respectively. Also , a car covered a distance

e. 780

Directions (90-91): What will come in place of the question


mark (?) in the following number series:
90. 11 10 18 51 200 ?
a. 885

b. 1025

c. 865

d. 995

the Ramans daughters present age and Ramans


mothers present age:
a. 45 years

b. 40 years

c. 50 years

d. cant determine

e. none

98. What is the value of twenty four percent of foure. none

ninth of five times square of twenty seven:


a. 388.8

b. 376.8

c. 378.6

d. 346.6

91. 25 48 94 186 370 ?


a. 738

b. 744

c. 746

d. 724

e. none

Directions (92 95) : In the following questions two


equations numbered I and II are given. You have to solve
both the equation and ___
Give answer (a) if x >y
Give answer (b) if xy
Give answer(c) if x <y

e. none

99. The sum of the circumference of a circle and the


perimeter of a square is equal to 272 cm. The
diameter of the circle is 56 cm. What is the sum of
area of circle and the area of the square:
a. 2464 sq. cm

b. 2644 sq. cm

c. 3040 sq. cm

d. cant determine

e. none of these
100. The largest and the second largest angles of a

Give answer (d) if x y

triangle are in the ratio of 4:3 respectively. The

Give answer (e) if x = y or the relationship cannot be

smallest angle is half the largest angle. What is the

established.

difference between the smallest and the largest

92. I. =

angles of the triangle:

II. + =

93. I.
II.

a. 30

b. 60

d. 20

c. 40

e. none

94. I. () +

II. + =
95. I. + =

Page | 12

ANSWERS: S.M.T- 2
(1-5):

E +A = O1-type of conclusion
Some flats are not houses (B)
No building is an apartment.

All apartments are flats.


E + A = O1 type of conclusion
Some flats are not buildings (C)
1.
3.

C
B

11. B; conclusion A is conclusion II


12. D;

2. A

(13-14):
Some seas are oceans
4.

(6-7):

5. D

All oceans are rivers

Ajay > Poonam > Ben

I + A = I-type of conclusion

Kim > Shreya

some seas are rivers (A)

Kim > Shreya > Ajay > Poonam > Ben


87%

70%

All oceans are rivers

65%

6.
7.
8.
9.

E
C
E;HI=J=KL<M
D; S > Q R < P
P > S: not true
10. A; K L =M > N
Now, N < K

No river is a canal.
A + E= E-type of conclusion
No ocean is a canal (B)
Some seas are rivers.

(11-15):

No river is a canal
All building are houses Universal Affirmative
I + E = O1-type of conclusion
(A-type)
(ii)
Some oceans are seas particular Affirmative
some canals are not seas (C)
(I-type)
13. A; All oceans are rivers.
(iii)
No house is an apartment Universal
Its converse some rivers are oceans, is true
Negative (E-type)
Thus, conclusion I is true
(iv)
Some houses are not apartments particular
14. E; conclusion B is conclusion I.
Negative (O-type).
Conclusion A is conclusion II.
(11-12):
15. D; No day is night
(i)

All buildings are houses

All nights are noon


E + A = O1- type of conclusion
some noon are not days (A)
All nights are noon.

No house is an apartment.
A + E = E-type of conclusion

No noon is an evening.
A+E = E-type of conclusion
No night is an evening (B)

No building is an apartment (A)


No house is an apartment
(16-20):
All apartment are flats

Page | 13

32.
33.
34.
35.
16. C; Two persons- P and S- are seated between Q and
T
17. B; Except S, all others are seated at the ends.
18. E; P and B are sitting exactly in the middle of rows
19. E; D and E are immediate neighbours of B. B sits
exactly in the middle of the row.
P faces B.
Either Q or S is an immediate neighbour of P, who
faces B.
20. D; R and A are sitting opposite diagonally.
21. E; ke stands for list
22. B; name ya; has la; been jo ; displayed si
(23-25):

C; The statement C would weaken the argument.


D; statement B is an inference.
A; Option (1) can be inferred.
B; statement D represents a reason behind curtailing
industrial development by some of the counties.
36. C
37. D
38. B
39. B
40. E
41. A
46. A
51. B
56. C
61. B

42. C

43. B

44. D

45. A

47. E

48. C

49. E

50. D

52. D

53. E

54. C

55. A

57. B

58. A

59. B

60. A

62. D

63. C

64. B

65. C

66. A; First S.P =

46000 88

= . 40480

100
40480 112

Second S.P =
= . 45337.6
After careful analysis of the given input and various steps of
100
rearrangement it is evident that words have been
Loss = Rs. (46000 45337.6 ) = Rs. 662.4
rearranged in the reverse alphabetical order and the 67. B; Third even number = 402 1 = 67 1 = 66
6
numbers have been rearranged in descending order. In the
Smallest
even
number
=
62
first step the word which will appear first in a dictionary is
Smallest number of set-B = 2 62 15 = 109
placed at the extreme left position and the highest number
Required sum = 109 + 110+111+112= 442
is placed at the extreme left position and the second
591
45
+ 780 =
highest number is placed at the extreme right position. 68. D; Total spent amount =
3
60
Similar rearrangement is carried on till we get the final
782 = . 7.82
108 5
output.
69. B; Speed of train = 108 kmph =
= 30 /
18
+280
Input : 49 last zen 16 82 yet can vast 33 aim 87 54
If the length of platform be x metre, then
= 30
Step I : aim 49 last zen 16 82 yet can vast 33 87 54
Step II : can aim 49 last zen 16 yet vast 33 54 87 82
Step III : last can aim 49 zen 16 yet vast 33 87 82 54
Step IV : vast last can aim zen 16 yet 33 87 82 54 49
Step V : yet vast last can aim zen 16 87 82 54 49 33
Step VI : zen yet vast last can aim 87 82 54 49 33 16
23. C ; six steps
24. E; this is step VI
25. E; none of these
(26-30):

12

x + 280 = 30 12 = 360 x = 80 metre

80
Mans speed =
= = 8 /

10
70. D; Let the three angles of quadrilateral be 13x, 9x and
5x respectively. 13x + 9x + 5x = 360 36 27x =
324
324 x =
= 12; required difference = 13x -5x = 8x
27
= 8 12 = 96
71. D; Number of girls:
6000 26
School C
900 = 1560 900 = 660
100
6000 29

School-E
1200 = 1740 1200 = 540
100
Required answer = 660+ 540 +600 = 1800
6000 9
72. C; Number of girls in School-B =
400 = 540
100
400
6000 29
Number of students in school E =
= 1740
100
Required Ratio = 900: 140: 1740 or 45: 7:87
6000 6
73. E; Required difference = 1200
= 1200
100
360 = 840
6000 9
74. B; number of students in school B =
= 540
75. E; number of girls in school A =

100
6000 12

220

26.
27.
28.
31.

100

-500 = 720 -

500 =220; required percentage =


100 = 41
540
76. E; Number of participants (athletics) from country C.
C; Except Harleen, all others are in the middle of the
Year 2006 (6.9 + 3.3)x 100 =1020
sides.
Year 2008 (6.6 + 4.2)x 100 = 1080
A; Bharat sits third to the left of Ali.
Year 2009 (7.9 + 6.3)x 100 = 1420
D; 29.D
30.B
Year 2010 (10.8 + 6.9)x 100 = 1770
C; Higher life expectancy in Southern India can be
ascribed to the genetic makeup of the population.

Page | 14

77. C ; Required average number of female athletes =


4.2+6.2+4.8+8.4+5.2+7.2
100 = 600
6

78. B ; percentage decrease =


79. B; Required percentage=

(6.94.8)
6.9
9.2

(11.4+8.4)

100 30
100 46

II.

= =

94. A; I. () + = ( ) + =

= = =
=

80. E; difference between the number if male and female


II.
+ = = =
participants:

Country A (6.6 4.2) 100 = 240


=
=

Country B (8.4 6.2) 100 = 220


95. A; I. + = + =
Country C (6.9 3.3) 100 = 360

= =
Country D (8.4 6.3) 100 = 210

II.
+ = + =
Country E (7.8 5.2) 100 = 260

+ = =
Calculations (81 85):

British passengers =
Females =

216
4

1200 18
100

96. B ; Speed of car =

= 216

Speed of Bus =

= 54

Males = 216 -54 = 162


Passengers from Madagascar =
Passengers from South Africa =

1200 6
100
1200 2
5

= 72 =

720
9

= 80

80 = 60

4
27

Speed of train = 60 = 108


15
Distance covered by train in 7 hours= 7 108 = 756
km.
97. C ; Let Ramans present age = x years

His daughters present age =


3
13

= 480

Males = 240 ; Females = 240


Indian Passengers= 1200 216 -72 -480 = 432
2

Females = 432 = 288 ; = 432 288 = 144


3

His mothers present age =

9
13
9 + 3 + 13
+ +
= 125
= 125 25
3
9
9
125 9
= 125 9 =
= 45
25
13

133
10
10
Required difference =
=
=
=
9
3
9
9
9
45 = 50
4
24
98. A; Required value = (27)2 5
= 388.8

81. B; Required ratio = 72: 240: 432 = 3:10:18


240
82. A; Required percentage =
100 111
216
9
100
83. A ; Average number of male passengers=
22
162 +72+240+144
618
99. C; circumference of circle = =
7
=
= 154.5
6
4
56 = 176 ; = 272 176 =
84. D ; Required difference = 144 72 = 72
96
96 = = 24 area of square =
85. E ; Total number of male passengers from Britain and
4
female passengers from India together = 162 + 288 =
24 x 24 = 576 sq. cm
22
450
= 2 =
28 28 = 2464 .
1898
2
2
7
86. C ;
72 = ? 13 ? 13 = 26 72 ? =
73
Required sum = (576 + 2464)sq.cm = 3040 sq. cm
144 = 12
100. C; The smallest angle of triangle is half of the largest
87. E;? = 72 24 2 (11)3 + 3 = 2352 1331 + 3 =
angle .
Ratio of three angles = 4:3:2 4x+3x+2x = 180 9x
1024 = 32
700 90
1000 50
= 180 x = 20 ; required difference = 4x 2x = 2x = 2
88. D;
?
+
170 630 + 500 170
o
100
100
x 20 = 40
960

89. D;?

; required

answer= 870
90. D; The pattern of the number series is:
11x 1 1 = 10 ; 10 x 2 2 = 18 ; 18 x 3 3 = 51 ; 51 x
4 4 = 200 ; 200 x 5 5 = 995
91. A; The pattern of the number series is:
25 x 2 2 = 48 ; 48 x 2 2 = 94 ; 94 x 2 2 = 186 ;
186 x 2 2 = 370 ; 370 x 2 2 =738
92. A ; I. = = =

= =

II. + = = =

93. C ; I. =

= =

Page | 15

AAA

BRIGHT ACADEMY

TM

CENTRE FOR EXCELLENCE IN EDUCATION

SBI MOCK TEST-3

How many meaningful English words can be made


with the letters LTSO using each letter only once in
each word:
a. None
b. One

Some planets are suns


Some stars are gases.
Conclusions:
I.
Some stars are planets
II.
Some suns are gases
III.
No moon is a planet
IV.
Some gases are moons
a. None follow
b. only I follow

c. two

c. only I and II follow

REASONING
1.

d. Three

e. more than three

Directions (2 3) : Study the number series given below


and answer the questions which follow:

e. only IV follow
6.

6 7 8 9 9 8 7 9 7 7 8 9 7 8 7 6 9 6 8 9 7 7 9 8
9 7 6 8 8 7
2.

How many such numbers are there in the given


series each of which when subtracted from the
following number, has a difference of 2:
a. Three
b. four
c.

3.

five

d. nine

e. None

Which of the following numbers is obtained when


th
18 number from the left of the number series is
th
added to 19 from the right:
a. 15
b. 20
c.

10

d. 17

e. None

Directions (4 8) : In each of the questions below are given


four statements followed by four conclusions numbered I,II,
III,IV . You have to take the given statements to be true even
if they seem to be at variance from commonly known facts.
Read all the conclusions and then decide which of them
given conclusions logically follows from the given statements
disregarding commonly known facts:
4.

5.

Statements:
Some robots are machines
Some computers are both robots and machines
Some animals are machines
Some toys are animals
Conclusions:
I.
Some toys are robots
II.
Some toys are machines
III.
Some animals are computers
IV.
Some robots are not toys
a. None follows
b. Only II and III follows
c. Only I and III follows
d. Only III follows
e. Only either I or IV follow
Statements:
All suns are stars
All moons are stars

d. only III follow

Statements:
All books are diaries
Some diaries are pens
Some pens are drawers
All drawers are chairs
Conclusions:
I.
Some drawers are diaries
II.
Some chairs are pens
III.
Some pens are books
IV.
Some diaries are books
a. None follows
b. Only II follows
c. only II and III follow

d. only II and IV follow

e. all follow
7.

Statements:
Some buildings are rivers
Some mountains are both buildings and rivers
Some roads are buildings
All roads are trucks
Conclusions:
I.
Some mountains are roads
II.
Some buildings are trucks
III.
Some rivers are roads
IV.
Some trucks are rivers
a. None follows
b. only I follows
c.

only II follows

d. only III follows

e. all follow
8.

Statements:
All tables are round.
Some hills are round.
Some rivers are hills
All rivers are conical.
Conclusions:
I.
Some rivers are round.
II.
Some hills are conical
III.
Some rivers are both hills and round.
IV.
Some tables are conical.
a.
None follows
b. only II follows
c.

only I and III follow

d. only II and III follow

Page | 1

Give answer (a) if the data in statement I alone are


Directions (9): Study the following information carefully and sufficient to answer the question, while the data in statement
II alone are not sufficient to answer the question.
answer the question.
A,B,C,D,E,F,G and H are sitting around a circle, facing the Give answer (b) if the data in statement II alone are
centre. A sits fourth to the right of H while second to the left sufficient to answer the question, while the data in statement
of F. C is not the neighbor of F and B. D sits third to the right I alone are not sufficient to answer the question.
e.

all follow

of C. H never sits next to G.


9.

Which of the following pairs sit between H and G:


a. BH
b. EF
c.

CE

d. DB

e. none

Give answer (c) If the data in statement I alone or in


statement II alone are sufficient to answer the question.
Give answer (d) if the data in both the statements I and II
are not sufficient to answer the question

Directions (10-12): In each of the following questions, two Give answer (e) if the data in both the statements I and II
rows of numbers are given. The resultant number in each together are necessary to answer the question.
row is to be worked out separately based on the following 13. How many marbles does Sanjay have:
rules and the question below the rows of numbers are to be
I.
Sanjay has 3 times more marbles than Ahmed.
answered. The operation of numbers progresses from the
II.
Ahmed has one third the number of marbles
which Suresh has who has as many marbles as
left to right.
Sanjay.
Rules:
14. How is Ramesh related to Lata:
(i)
If an odd number is followed by another
I.
Yogendra is Latas Father.
composite odd number, they are to be
II.
The only sibling of Rameshs maternal aunt is
multiplied.
married to Yogendra.
(ii)
If an even number is followed by an odd 15. IF World peace is important is written as ni pe la
number, they are to be added.
so in a code language, what is the code for peace:
(iii)
If an even number is followed by a number
I.
an important call is written as ka so ba
which is the perfect square, the even number is
II.
this is world map is written as de la ni ru
to be subtracted from the perfect square.
16. In which direction is Neha walking:
(iv)
If an odd number is followed by a prime odd
I.
Neha walked 15 kms from the starting point
number, the first number is to be divided by the
and took a left turn.
second number.
II.
She was walking towards north before taking a
(v)
If an odd number is followed by an even
left turn.
number, the second one is to be subtracted from 17. What was the day of the week on fifteenth of the
the first one.
month:
10. 14
11
9
I.
The first Sunday of the month was sixth.
104
q
11
II.
The last day of the month was thirty first.
If q is the resultant of the first row , what is the
Directions (18 19): Below in each question are given two
resultant of the second row:
statements A and B. These statements may be either
a. 1331
b. 132
independent causes or may be effects of independent
c. 5
d. 11
e. none
causes or a common cause. One of these statements may
be the effect of the other statement. Read both the
11. 36
27
7
statements and decide which of the following answer
m
15
124
If m is the resultant of the first row, what is the choices correctly depicts the relationship between these two
statements.
resultant of the second row:
a. 125
b. 25
Mark answer (a) if statement A is the cause and statement
B is its effect.
c. 11
d. 15
e. none
12. 22
27
7
7
9
14
nd
What will be the answer if the resultant of the 2
row is divided by the resultant of the first row:
a. 7
b. 14

c.

63

d. 48

e. none

Mark answer (b) if statement B is the cause and statement


A is its effect.
Mark answer (c) if both the statements A and B are
independent causes.
Mark Answer (d) if both the statements A and B are effects
of independent causes.

Directions (13 17): Each of the questions below consists Mark Answer (e) if both the statements A and B are effects
of a question and two statements numbered I and II given of some common cause.
below it. You have to decide whether the data provided in 18. Statement A: School Education has been made free of
the statement are sufficient to answer the question. Read
cost for girls.
both the statements andStatement B: Literacy rate among the girl child has
been steadily improving.

Page | 2

19. Statement A: Hallmarking of gold ornaments was


introduced all over the country.
Statement B: Many customers do not prefer to buy
Hallmark jewellery

b.
c.
d.
e.

The age of C must be 26 yrs


Only C,D and F are elder than E
D is 23 yrs old
None is true

Directions (20 24): Study the following information Directions (27 31) : Study the following information
carefully and answer the questions given below:
carefully and answer the questions which follow:
There are eight people G,H,I,J,K,L,M and N- each of them
belongs to different states viz. Assam, Bihar, Karnataka,
Maharashtra, Punjab, Rajasthan, Tamil Nadu and West
Bengal, but not necessarily in the same order. They live in
an eight story building, having floors numbered one to eighteach of them on different floor. ( The Ground Floor is
numbered floor No.1 , the floor above it floor no.2 and so
on).G lives on even numbered floor. Three people live
20. Which of the following men is third tallest of the between J and the Person from Assam. Three persons live
between L and G. The person from Assam live immediately
five:
below Gs floor. N lives immediately above Gs floor and
a. Sourav
b. Rahul
immediately below Ks floor. G is form Tamil Nadu. There
c. Harish
d. Kunal
are five floors between the floors on which J and the person
from Karnataka live. The person from Wet Bengal lives on
e. Kunal or Rahul
the floor immediately below the floor on which the person
21. Who among the following men is tallest:
from Assam lives. H lives on the even numbered floor. I does
a. Sourav
b. Rahul
not belong to Assam. Three persons live between the
c. Kunal
d. Harish
persons from Bihar and West Bengal .Four persons live
between the persons from Bihar and Maharashtra. L does
e. Sourav or Harish
not belong to Rajasthan.
22. If the five men are made to stand in a line according
to the height, first in ascending order, then in 27. On which of the following floors does the person
from Bihar live:
descending order, then whose position will remain
a. Fourth
b. Sixth
the same in both the arrangements:
a. Harish
b. Rahul
c. Fifth
d. eighth
e. none of these

There are five men Anuj, Kunal, Sourav, Rahul and


Harish. The one who is tallest is not the youngest.
Kunal is older than only Harish. Sourav is older than
Rahul but shorter than him. Only one person is taller
than Rahul. Anuj is shortest while younger than only
Sourav and Rahul. Only two men are shorter than
Sourav.

c. Kunal

d. Sourav

e. Rahul or Anuj

28. Who lives on the top most floor:


a. K
b. G

c. N
d. M
e. none of these
23. Who among the following is older than Kunal but
29. To which state does J belong:
younger than Rahul:
a. West Bengal
b. Karnataka
a. Harish
b. Sourav
c. Anuj

d. Data inadequate

e. Sourav or Anuj
24. Who among the following is taller than Anuj only:
a. Sourav
b. Harish

c. Rahul

d. Kunal

e. Kunal or Harish
Directions (25 26) : Study the following information
carefully and answer the questions given below:

c.

Rajasthan

d. Assam

e. Punjab

30. Who lives exactly between H and L:


a. M
b. I

c.

d. N

e. K

31. The person from the state of Assam lives on which


floor:
a. Second
b. Third

c.

Sixth

d. Fourth

e. Fifth

32. A tour and travel agency was sued by three


customers as it had promised a discount on the
holiday package but when examined later, it was
found that the agency had actually charged 12
percent more than what other agencies were
offering.
Which of the following is an inference which can be
25. What would possibly be the age of B:
made from the facts stated in the above paragraph:
a. 18 yr
b. 26 yr
a. Cheating may be a ground for filing a case.
c. 22 yr
d. 16 yr
e. 28 yr
b. It is not a grave crime to charge extra money for
holiday package
26. Which of the following statements may be true on
c. Different tour and travel agencies charge
the basis of information given above:
differently.
a. E may be 20 yr old

There are six people- A,B,C,D,E and F each of them


having different ages. D is elder than A and B. F is
elder than E only . A is elder than B.C is elder than D. If
the age of F is 19 years, then the third eldest person is
of 24 years.

Page | 3

d. It is not essential to offer what has been promised could happen in countries where lenders dont have to
earlier.
disclose interest rates. When a Mexican micro financier went
public, revealing its loans had rates of about 86% annually,
e. None of these
Directions (33 35): Study the following information the Consultative Group to Assist the Poor (CGAP) criticized
it for putting shareholders ahead of clients. The pressure of
carefully and answer the questions given below:
turn a profit also forces micro financiers to change their
In a certain code languagebusiness models in ways that depart from the industrys
core mission: to help poor people lead better lives. Such
he needs some help is written as tu gi ja le
shifts had caused the average loan size to triple. Moreover,
needs medicine and some is written as le da tu si
a lower percentage of loans go to women because they tend
to take out smaller sums. According to CGAP, with the flood
some know now help is written as tu pa gi ki
of new large percentage of cross-border funds go to Latin
now and where go is written as jo ze ki si
America and eastern Europe, the worlds most developed
33. What does da tu ja mean in the given code microfinance markets. The poorest of the worlds poor, who
are predominantly in Asia and Africa get left out, says the
language:
CEO of the nonprofit Grameen Foundation, Which helps
a. Now help medicine
develop microfinance institutions.
b. He some medicine
c. He now medicine
Segmenting the industry, might be worthwhile if it allows
d. He needs medicine
more of the poor to get access to credit .Multinational
e. Cant determine
corporations could take the top microfinance institutions to
34. What is the code for he know where in the given the next level, and the remainder could be the responsibility
code language:
of development groups and regional banks. Yet making
a. Ja pa jo
b. pa ze ja
loans to poor people is hardly a poverty cure. Poverty rights
and the rule of law matters too. One cannot overidealize
c. da le ze
d. tu gi si
what microfinance alone can do. Most nonprofits started with
e. cant determined
lending simply because local laws prohibited nonbanks from
35. H is the father of J. J is the brother of L. N is the offering deposit accounts. With an increase in competition
wife of L.K is the son of N. L and J are children of and marketing efforts, poverty- alleviation experts are
H. P is the wife of H. K and Z are grandchildren of P. concerned that people will be talked into loans they wouldnt
otherwise want, For example, organizations like Mibanco are
Who is the uncle of K:
providing consumer loans. There is nothing wrong with
a. J
b. H
buying TVs and microwaves on credit, but certain markets,
c. Z
d. Cant determined
like Mexico, have been flooded with loans that have nothing
e. none of these
to do with providing capital to aspiring entrepreneurs just
increasing household debt.

ENGLISH LANGUAGE

Directions (36 45): Read the following passage carefully


and answer the questions given below it. Certain words have
been printed in bold to help you locate them while
answering some of the questions.
Giving loans to impoverished women to make ceramics or to
farmers to buy milk cows were not seen as a great business
Microfinance was an industry championed by antipoverty
activists. Today it is on the verge of a revolution, with billions
of dollars from big banks, private-equity shops and pension
funds pouring in, driving growth of 30% to 40% this year
alone. In 1998, a non-profit microfinance organization in
Peru, converted into bank (called Mibanco).This
demonstrated that the poor are good risks who repay loans
on time and getting them together, not only chips away at
poverty but also turns a profit. The success of Mibanco has
piqued the interest of commercial banks, which had
previously shunned the countrys poor. Now big banks are
going after Mibancos clients with low-rate loans and
realizing it takes special know-how to work with the
unbanked- are hiring away Mibancos staff.
But with the emergence of players who are only out for profit,
microfinance schemes could end up milking the poor. This

36. What is CGAPs fear with respect to new entities


providing microfinance:
a. NGO will be unable to survive in an environment of
cut throat competition.
b. The poor will hesitate to take advantage of credit
facilities because of the formalities involved.
c. The poor in the developed world will be overlooked.
d. The interests of the most deserving among the poor
will be neglected.
e. Shareholders interest will be ignored.
37. What is the authors opinion about the competition
for customers among micro financiers:
a. It benefits the poor by providing them with loans
would have otherwise not had access to.
b. It is futile since the poor have to pay high rates of
interest on property loans.
c. It is not beneficial since firms waste their profits on
marketing rather than helping the poor.
d. It is a disadvantage since micro financiers use any
means possible to recover loans.
e. None of these
38. Which of the following is/are challenge/s faced by
Mibanco at present from big banks:
A. Ensuring loyalty of their customers
B. Retention of employees
C. Maintaining low interest rates.

Page | 4

a. only C

b. Both A and C

c. both B and C

d. Only B

e. none of these
39. Which of the following is/are consequence/s of
micro financiers altering their business models:
A. Larger loan amounts get sanctioned.
B. Debt among the poor has fallen in some countries.
C. Drop in the loans awarded to women.
a. Both A and B
b. both A and C

c. only C

d. All A,B,C

e. none of these
40. Which of the following cannot be said about the
Grameen Foundation:
A. It regulates the activities of microfinance firms in
developing countries.
B. It functions primarily in Asia and Latin America
C. It approves of privatizing microfinance institutions.
a. Both A and B
b. Only B

c. both A and C
d. All A,B,C e. none
Directions (41 43) : Chose the word/group of words
which is most similar in meaning to the word/group of
words printed in bold as used in the passage.
41. Depart
a. Absent

c. divide
42. Piqued
a. Provoked

c.

disturb

43. Verge
a. Tend

b. retirement

d. expire

e. vary

b. irritated

d. offended e. fascinated

47. After we hear the news of the floods, we immediately


decided to join in the relief efforts.
a. Since we have hear
b. While we were heard
c. On hearing
d. Upon hearing that
e. no correction required
48. With no rain and any access to clean water many
farmers will lose their crops.
a. Without any rain or
b. Without rain and nor
c. No raining and no
d. There is no rain and no
e. no correction required
49. Her goal to win a gold medal and made her country
proud.
a. For making her country proud
b. Made her countrys pride
c. By making her country proud.
d. Make her country proud
e. no correction required
50. Today a kilo of rice is near five times more expensive
than it was in 2003:
a. Near to five times
b. Nearly five times
c. Nearly by five times
d. Nearing five times
e. no correction required
Directions (51 55): Read each sentence to find out
whether there is any grammatical error or idiomatic error in
it. The error, if any, will be in one part of the sentence. The
number of that part is the answer. If there is no error, the
answer is (e). (Ignore errors of punctuation, if any)

51. Wheat productions in the region (a)/ will fall


drastically (b)/ unless we do anything (c)/ to stop
climate change. (d)/ no error (e)
c. ascent
d. slope
e. threshold
52. The Indian stock market is (a)/ one of the worse (b)/
Directions (44 45): choose the word/group of words
performing stock markets (c)/ in the recent times.
which is most opposite in meaning to the word/group of
(d)/ no error (e)
words printed in bold as used in the passage
53. With inflation at eleven per cent (a)/ companies
44. Aspiring
need to come up (b)/ with innovative ways to (c)/
a. Uninterested b. ungrateful
get customers to buy its goods (d)/ no error (e)
54.
Private companies which profits (a)/ have grown
c. anxious
d. miserable e. Fraudulent
due to (b)/ the high price of oil (c)/ should offer
45. Core
discounts on cooking gas. (d)/ no error (e)
a. Clear
b. unnecessary
55. Having work in (a)/ both public and private sector
banks (b)/ Mr. Rao is the ideal choice to (c)/ take
c. crust
d. beside
e. uncoordinated
over as chairman. (d)/ no error (e)
Directions (46 50): Which of the phrases (a),(b),(c),(d)
given below each sentence should replace the word/phrase Directions (56 90): In the following passage there are
printed in bold in the sentence to make it grammatically blanks, each of which has been numbered. These numbers
correct? If the sentence is correct as it is given and no are printed below the passage and against each. Five words
are suggested , one of which fits the blank appropriately.
correction is required, mark (e) as the answer
Find out the appropriate word in each case.
46. Most people feel that the most important reason to
Without doubt there is one thing 56 to all of us we have all
recycle is its impact on future generations.
played a game at some time in our lives. Most of us play to
a. Are their impacts
b. has an impact
relax or have fun, but for many , playing a game or a sport is
c. it is impact
d. have an impact
a way to 57 poverty behind. In fact, in many African
countries, playing a sport professionally can 58 the lives of a
e. no correction required
persons entire family.
b. crossroad

Page | 5

For example, in the small town of Bekoji, in Ethiopia 59 than 69. Number of girls speaking only Hindi is what percent
of the total number of students speaking only
a hundred boys and girls can be seen running at dawn every
day. Each of these youth is 60 and serious and their coach is
Hindi:
61 that one of them will be a world champion. This seems
a. 38.2
b. 71.8
like an idle 62 but it is virtually a guarantee in this small
c. 31. 2
d. 78
e. none of these
community 63 mainly farmers. Many of the fastest male and
70.
What
is
the
respective
ratio
between
the number of
female distance runners in the world hail from this small
boys
and
girls
speaking
both
the
languages:
town. A small hand-painted sign which greets visitors outside
a. 23 : 25
b. 12 : 25
Bekoji 64 Welcome to the Village of Athletes. Children here
start running at an early age, 65 great distances to fetch
c. 12 : 13
d. 25 : 13
e. none of these
water and firewood or to reach school. At the Olympics,
2
runners from this small town are likely to win more medals 71. Area of the circle is 616 cm . What is the area of the
rectangle? (dot indicates centre of the circle).
than those from developed countries. It will give their families
a way out of poverty.
56. a. popular
c. common
57. a. leave
c. forgot
58. a. shift
c. arrange
59. a. larger
c. more
60. a. performed
c. rival
61. a. confident
c. optimist
62. a. precaution
c. suspicion
63. a. for
c. that
64. a. wish
c. inform
65. a. competing
c. driving

b. accepted
d. alike
e. similar
b. alleviate
d. prevent
e. reduce
b. changes
d. control
e. transform
b. further
d. greater
e. over
b. concentrated
d. focused
e. playful
b. convince
d. intended
e. privilege
b. boast
d. risk
e. worship
b. existing
d. comprising e. consisting
b. warn
d. Notices
e. Reads
b. covering
d. Measuring e. Following

a. 784 cm

b. 196 cm

c.

d. cant determine

392 cm

e. none

72. The product of two consecutive even numbers is


9408 .Which is the greater of the two numbers:
a. 96
b. 98

c.

94

d. 92

e. none of these

73. What is 27% of 36% of :

a. 239
b. 241

c. 243

d. 245

e. none of these

74. The simple interest accrued on an amount of Rs.


40,000 at the end of three years is Rs. 12,000. What
would be the compound interest accrued on the
same amount at the same rate in the same period:
Directions (66 70) : These questions are based on the
a. Rs. 18,765
b. Rs. 15,350
following data. Study it carefully and answer the questions
c. Rs. 21,555
d. Rs . 13,240
that follow:

QUANTITATIVE APTITUDE

e. none of these
In a school having 400 students boys and girls in the ratio of
3 : 5 respectively. The students speak Hindi , English or both 75. If the numerator of a fraction is increased by 300%
the languages. 12% of the boys speak only Hindi, 22 % of
and the denominator is increased by 100%,the
the girls speak only English. 24% of the total students speak

resultant fraction is .What was the original


only Hindi and the number of boys speaking both the

fraction:
languages is six times the number of boys speaking only
15
13
Hindi.
a.
b.
19
19
66. Total how many boys speak Hindi:
5
4
c. 9
d. 9
e. none of these
a. 18
b. 126
76. 42 women can do a piece of work in 18 days. How
c. 108
d. 26
e. none of these
many women would be required to do the same
67. How many girls speak only Hindi:
work in 21 days:
a. 55
b. 117
a. 36
b. 24
c. 96

d. 78

e. none of these

68. Total how many students speak English:


a. 304
b. 79

c. 225

d. 117

e. none of these

c.

30

d. 44

e. none of these

77. The ages of Sulekha and Arunima are in the ratio of


9 : 8 respectively. After 5 years the ratio of their
ages will be 10 : 9. What is the difference in years
between their ages:

Page | 6

a.

4 yr

c.

6 yr

number of students studying B.Com in all colleges


together:
a. 39
b. 21

b. 5 yr

d. 7 yr

e. none of these

78. By how much is of 568 lesser than of 1008:

a. 444
b. 448

c. 44

d. 33

e. 17

85. Number of students studying B.A in college B


forms what per cent of total number of students
studying all the disciplines together in that college:
79. Vishakha spent Rs 68,357 on the renovation for her
(rounded off two digits after decimal)
home, Rs. 25,675 on buying music system and the
a. 26.86
b. 27.27
remaining 28% of the total amount she had as cash
with her. What was the total amount:
c. 29.84
d. 32.51
e. none of these
a. Rs. 94,032
b. Rs. 36,568

c.

452

d. 456

c.

Rs. 1,30,600

e.

none of these

e. none of these

d. cant determine

Directions (86 90) : Study the table carefully to


answer the questions that follow:

80. Five-sixth of a number is 720. What will 45% of that


number be:
a. 346.6
b. 388.8

c.

392.2

d. 344.4

e. none of these

Directions (81 85): Study the following graph


carefully to answer the questions that follow:
Number of students enrolled in three different
disciplines in five different colleges
B.A

B.Sc.

B.Com
86. What is the respective ratio of total number of men
selecting product B to those selecting product E:
a. 69 : 49
b. 7:5

500
450
400
350
300
250
200
150
100
50
0

c.

208 : 147

d. 1040 : 739 e. none of these

87. Number of men selecting product C forms what


percent of those selecting product F: (rounded off
to two digits after decimal)
a. 21.05
b. 475.04

c. 25.56

d. 460.08

e. none of these

88. Total number of women selecting product E forms


A
B
C
D
E
approximately what percent of the total number of
people selecting all the products together:
a. 11
b. 15
81. What is the total number of students studying B.Sc.
in all colleges together:
c. 20
d. 2
e. 7.5
a. 1825
b. 1900
89. What is the total number of children selecting
c. 1650
d. 1775
e. none of these
product A:
a. 14085
b. 5337
82. What is the respective ratio of total number of
students studying B.Sc. in the college C and E
c. 20031
d. 16389
e. none of these
together to those studying B.A. in the colleges A
90. What is the approximate average number of women
and B together:
selecting all products together:
a. 24 : 23
b. 25 : 27
a.
16707
b. 16686

c.

29 : 23

d. 31:25

e. none of these

c.

16531

d. 16429

e. 16311

83. What is the respective ratio of total number of


students studying B.Sc. ,B.A and B.Com. in all the Directions (91 92): What approximate value should
come in place of question mark (?) in the following
colleges together:
a. 76 : 72 : 77
b. 67 : 71 : 75
questions : (You are not expected to calculate the

c. 71 : 68 : 75
e. none of these

d. 75 : 71 : 68

84. Number of students studying B.Com in college C


forms approximately what percent of the total

exact value)
91.

(2198 1347 403) (159 113-27)= ?


a. 15
b. 24

c.

37

d. 49

e. 53
Page | 7

92.

. =?
. 1200
b. 1000
. 1600
d. 1400

e. 800

Directions (93 95): What should come in place of question


mark(?) in the following number series:
93. 40280625
a. 0.248

c.

0.424

732375 16275 465 18.6 1.24 ?


b. 0.336

d. 0.512

e. 0.639

94. 14 12 21 59 231 1149 ?


a. 6987
b. 6787

c. 6887

d. 6687

e. 6587

95. 3 6 21 28 55 66 ? 120
a. 103
b. 104

c.

108

d. 106

e. 105

Directions (96 100): Each question below is followed by


two statements A and B. You are to determine whether the
data given in the statement is sufficient for answering the
question. You should use the data given in the statement is
sufficient for answering the question. You should use the
data and your knowledge of mathematics to choose between
the possible answers:
Give answer (a): if the data in statement A alone are
sufficient to answer the question, while the data in statement
B alone are not sufficient to answer the question
Give answer (b): if the data in statement B alone are
sufficient to answer the question, while the data in statement
A alone are not sufficient to answer the question.
Give answer (c) : If both the statement A and B together are
needed to answer the question
Give answer (d) : If either the statement A alone or
statement B alone is sufficient to answer the question
Give Answer (e) : If you cannot get the answer from the
statement A and B together, but need even more data
96. Is A an odd number:
A. A multiplied by an odd number is equal to an odd
number.
B. A is not divided by 2.
97. The ages of pradumn and Gunit are in the ratio of 7
: 5. What is the age of Pradumn.
A. The ages of Pradumn and Nandini are in the ratio
of 3:1
B. After 7 yrs the ratio of Pradumns and Avirals ages
will be 4 : 3.
98. What is the salary of B, in a group of A,B,C and D
whose average is Rs. 62,880:
A. Total of the salary of A and C is exact multiple of 8.
B. Average of the salary of A ,C and D is Rs. 61,665.
99. What is the three digit number:
A. The three digit number is divisible by 9.
B. The first and the third digit is 6.
100. What is the profit earned by selling a printer for Rs.
3,000:
A. The cost price of 6 such printers is equal to selling
price of 5 such printers
B. 20% profit is earned by selling each printer.

Page | 8

AAA

BRIGHT ACADEMYTM

CENTRE FOR EXCELLENCE IN EDUCATION

ANSWERS: SBI.M.T-3
1.
2.
3.

C; Meaningful words: SLOT, LOST


B; 678998 79 778978769 68 97 79 897 68 87
th
th
A; 18 number from the left = 6 and 19 number from
the right = 9 therefore 6 + 9 = 15

(4 8):
(i) All suns are stars Universal Affirmative (A-type)
(ii) Some robots are machines particular affirmative
(I-type)
(iii) No moons is a planet universal negative (E-type)
(iv) Some moons are not planets particular Negative
(O-type)
4. E; All the four premises are particular Affirmative. No
conclusion follows from the two particular premises.
9. C
Conclusions I and IV form complementary pair.
10.
D;
14
+
11
= 25 (rule ii)
Therefore, either conclusion I or IV follows.
25

9
=
225 (rule i)
5. B; Some planets are suns.
225 104 = 121 (rule iii)
121 11 = 11 (rule iv)
All suns are stars.
11.
C;
36
+ 27 = 63 (rule ii)
I + A I-type of conclusion
63
7 = 9 (rule iv)
Some planets are stars
9

15 = 135 (rule i)
Conclusion I is converse of this conclusion.
135
124 = 11 (rule v)
6. D; Some pens are drawers
12. A; 22 + 27 = 49 (rule ii)
49 7 = 7 (rule iv)
All drawers are chairs
7 9 = 63 (rule i)
I + A I-type of conclusion
63 14 = 49 (rule v)
some pens are chairs
49 7 = 7
Conclusion II is converse of this conclusion.
13.
D
Conclusion IV is the converse of the first premise.
14. D
7. C; Some buildings are roads
15. E; From both the statements.
world peace is important ni pe la so
All roads are trucks.

8.

I + A I-type of conclusion.
some building are trucks
This is conclusion II.
B; Some hills are rivers

an important call ka so ba
this is world map de la ni ru
The code for peace is pe
16. E;

All rivers are conical.


I + A I-type of conclusion
some hills are conical
This is conclusion II.
(9): Sitting arrangement
Neha is now walking towards west.
17. A ; From statement I.
st
1 of the month Tuesday
th
8 of the month Tuesday
th
15 of the month Tuesday

Page | 9

18. A; Statement A is the cause and Statement B is its Boys who speak only English = 150 108 18 = 24
effect.
66. B 67. D 68. A 69. E 70. C
19. D
22
71. C; r 2 = 616
r 2 = 616 r = 14 cm
7
(20 24) According to age
Length of rectangle = 28 cm and breadth = 14 cm
Area of rectangle = 28 x 14 = 392 sq. cm
Sourav > Rahul > Anuj >Kunal > Harish
72. B; From the given alternatives , 98 x 96 = 9408
According to height:
5
36
27
73. C; Required answer = 4500

= 243

Kunal > Rahul > Sourav > Harish > Anuj

20. A 21. C

22.D

23. C

24. B

(25 26)
D>A,B;F>E;A>B;C>D
Now ; C > D > A > B > F > E
24 yr
25. C
(27 31)

26.B

19 yr

74. D ; =

100

9
100
12000 100
40000 3

100

= 10%


= +
= +

= . = .

75. A; Let the original fraction be .


According to the question,
15

400

200

30
19

30 2
4 19

19

76. A; Days
Women
18
42
Floor No. Person
State
21
x

21
:
18
:
:
42
:
x
8
K
Bihar
42 18
=
= 36
7
N
Karnataka
21
77. B; Let the present ages of Sulekha and Arunima be 9x
6
G
Tamil Nadu
and 8x yrs respectively.
According to the question; After 5 yrs,
5
M
Assam
9 + 5 10
=
81 80 = 50 45 = 5
8
+5
9
4
H
West Bengal
Required difference = 9x 8x = x = 5 yrs
7
3
3
I
Maharashtra
78. D; Required difference = 1008 568 = 882
8
4
426 = 456
2
L
Punjab
79. C; Let the total amount be Rs.x .
1
J
Rajasthan
According to the question, Percentage of spent
amount = 100 28 = 72%
27. D 28. A 29. C 30.B
31. E
Amount spent = Rs (68357 + 25675 ) = 94032
32. A;
72

= 94032 = . 130600
33. B
34.E
100
35. A; P is the mother of J and L. K is the son of L and N. 80. B; Let the number be x.
J is the brother of L. Therefore, J is the uncle of K.
5
According to the question, = 720 = 864
6
36. D 37. E 38. A
864 45
45% 864 =
= 388.8
41. A 42. E
39. C 40. D
100
81. B; Total number of students studying B.Sc. in all the
43. E 44. A 45. B 46. E 47. C 48. A
colleges together= 375+350+325+400+ 450 = 1900
49. D 50. B
82. D; Total number of students studying B.Sc. in Colleges
51. C 52. B 53. D 54. A 55. A
C and E = 325+450= 775
Total number of students studying BA in Colleges A
56. C 57. B 58. E 59. C 60. D 61. A
and B = 300+325= 675
62. C 63. D 64. E 65. B
Required Ratio = 775: 675 = 31: 25
83.
A; Total number of students studying in different
Calculations (66 70)
steams in all the colleges:
Number of boys = 150, Number of girls = 250
B.Sc. = 1900 ; BA = 1800 ; B.Com = 1975
12 150
Required ratio = 1900:1800:1975 = 76:72:77
Boys who speak only Hindi =
= 18
100
84. E; Number of students studying B.Com in college C =
Boys who speak Hindi and English = 18 x 6 = 108
325
22 250
Total number of students studying B.Com= 1875
Girls who speak only English =
= 55
350
100
Required percentage =
100 17
1975
400 24
Girls who speak only Hindi =
18 = 96 18 = 78
85.
E;
Total
number
of
students
in college B = 325 + 350 +
100
475
=
1150
Girls who speak Hindi and English = 250 78 55 = 117
Number of students studying BA in college B = 325

Page | 10

325
A number is divisible by 9 if sum of its digits is
Required ratio =
100 = 28.26
1150
divisible by 9.
86. C; Total number of men selecting product B = 39% of
100. D ; From statement A, Let CP of 1 printer = Re.1
39
36800 = 36800
= 39 368
CP of 5 printers = Re. 5 and SP of 5 printers =
100
Total number of men selecting product E = 48300
Rs.6
21
1
= 483 21
Gain % = 100 = 20%
100
5
100
Required ratio = 368 x 39 : 483 x 21 = 208 : 147
CP =
3000 = . 2500
120
56340 45
87. A; Number of men selecting product C =
=
Gain = Rs (3000 2500) =Rs 500
100
25353
From statement B, We can also find the answer.
Number of men selecting Product F = 35580
15
= 5337;
100

5337

Required percentage =
100 = 21.05
25353
88. E ; Number of women selecting product E=48300
44
= 21252
100
Total number of people selecting all the products
together = 45525 + 36800 + 56340 + 62350 +
48300+ 35580 = 284895
21252
Required percentage =
100 7.5
284895
89. D; Number of children selecting product A = 45525
36
= 16389
100
90. A; Total number of women selecting all the products
44
33
together = 45525
+ 36800
+ 56340
30

100

28

44

100

35

+ 62350
+ 48300
+ 35580
=
100
100
100
100
20031 + 12144 + 16902 + 17458 + 21252 + 1453 =
100240
100240
Required average =
= 16707
91. B; ? =

= .

92. D ; ? = . . .
Nearest answer = 1400
93. A; The given number series is based on the following
pattern:
40280625 55 = 732375 ; 73237545 = 16275 ;
16275 35 = 465 ; 465 25 = 18.6 ; 18.6 15 = 1.24 ;
? = 1.24 5 = 0.248
94. C; The given number series is based on the following
pattern:
14 x 1 -2 = 12 ; 12 x 2 -3 = 21 ; 21 x 3 4 = 59 ; 59 x 4
5 = 231 ; 231 x 5 6 = 1149 ; ? = 1149 x6 7 =
6887
95. E ; The given number series is based on the following
pattern:
3
6
21
28 55
66 ?
12

96.
97.
98.

99.

+3
+7
+11
+15
? = 120 -15 = 105
D; From statement A, 3 x 5 = 15 ; 5 x 9 = 45 (an odd
number) , It is also obvious from statement B.
E;
B; A + B + C + D = Rs (4 x 62880)
From statement B, A + C + D = Rs.(3 x 61665)
Bs salary = (A+B+C+D)s salary (A+C+D)s
salary.
C ; From statement A, The three digit number is
divisible by 9.
From statement B, Number = 6 * 6

Page | 11

Vous aimerez peut-être aussi